Download as pdf or txt
Download as pdf or txt
You are on page 1of 84

Get more from: MplusX Qbank (https://member.mplusx.

com/)
Facebook Page: (https://www.facebook.com/mplusxqbank/)

Patient with HTN, DM, was on warfarin, perindopril, statin, metformin. 1wk ago started
amiodarone. Now presents with right thigh swelling,temp 37.9 ,tender, red thigh is 4cm
bigger.
a. aterial embolus

b. cellulitis

c. dvt

d. drug interaction

Lady starting permanent night shift. Has balanced food, used to do outdoor activity,
what to do?

a. Vit d supplement
b. Ask to come after 6 months
c. Advice diet plan
d. Reassure

38 yrs old lady wearing burqa visited gp for cervical screening. What else would you
screen?
a. Br ca
b. Skin
c. Vit d
d. Fbs

AfraTafreeh.com
Get more from: MplusX Qbank (https://member.mplusx.com/)
Facebook Page: (https://www.facebook.com/mplusxqbank/)

Similar ECG (AF) given, 65years old man with only hypercholesterolemia, asked long
term management :

a. Asprin

b.Warfarin

c.Digoxin

d.Metoprolol

AfraTafreeh.com
Get more from: MplusX Qbank (https://member.mplusx.com/)
Facebook Page: (https://www.facebook.com/mplusxqbank/)

Chest x ray showing patchy infiltrates and the patient was a chronic smoker n had long
history of cough now came with 2 weeks history of rusty sputum and fever asking
diagnosis :

a.Bronchiectasis

b. Pulmonary Fibrosis

c. COPD

d. Acute on chronic bronchitis

e. Lung cancer

A 7 years old girl with soiling in her underwear, she tells lies about accidents at school
and hide her soiled underpants and cry and quarrels with her brother when he calls her
smelly. What is your diagnosis?

a. Regression

b. Conduct disorder

c. Oppositional Defiant Disorder

d. Delayed milestones

e. Depression

https://childpsychologist.com.au/encopresis-soiling-in-school-aged-children/

Woman 55 years came to doctor for regular medical check up.Maternal aunt has breast
CA at --- year . What to check her?

a.Colposcopy 2 yrly

b.5 yearly cervical cancer screening

c.5 yearly colon cancer screening

AfraTafreeh.com
Get more from: MplusX Qbank (https://member.mplusx.com/)
Facebook Page: (https://www.facebook.com/mplusxqbank/)

d.mammogram 2 yearly

Patient wants to be your friend on social media what you should do?

a. Tell them they can discuss only medical related issues


b. They can message you directly
c. Accept the request but don’t contact directly
d. Decline the request
e. Delete your social media site

16 years old girl came to you besides contraception and safe sex what advice you would
like to give her?

a.Check chlamydia 6 monthly

b.Do all STDs 12 monthly

c.Cervical cancer screening from age 25years

d.Advise her to take HPV booster in 5 year

83 years man known case of COPD with 20 cigarettes per day since 18 years of age.
Tried to stop 4 times during last year. smoke first cigeratte within 10 mins after wake
up. Have strong family history of smoking. What factor is important indicator to stop
smoking or to start nicotine replacement therapy?
a.Family history

b.Duration of smoking

c.History of cessation

AfraTafreeh.com
Get more from: MplusX Qbank (https://member.mplusx.com/)
Facebook Page: (https://www.facebook.com/mplusxqbank/)

d.Smoking related pulmonary disease

e.First cigeratte time

6 weeks old baby with continuing jaundice, . Asking cause

a. Biliary atresia
b. Hypothyroid
c. Breast milk jaundice
http://americanpregnancy.org/breastfeeding/breastfeeding-and-jaundice/

52 years old female comes for screening. Her mother is diagnosed with Ca colon at the
age of 68 years old and she is otherwise well . What will you do for her?

a.No further screening test is necessary

b.Fobt 2 yearly

c.colonoscopy

d.sigmoidscopy

man recently migrate to Australia married say that people in his country are after him
to kill him. what to check in the collateral hx to reach Dx?

a.family hx of MOOD disorder

b.Low socialblity in the primary school

c.immigration visa status

d.the truth of his fear

AfraTafreeh.com
Get more from: MplusX Qbank (https://member.mplusx.com/)
Facebook Page: (https://www.facebook.com/mplusxqbank/)

e. Hx of physical trauma

diabetic pt, non healing ulcer, pic 2 ulcers in medial malleolus, and foot pulses not
pulpable. X ray already done. what to do for Mx?

a. blood culture

b. white cell scan

c, swab from ulcer

d. arterial dupplex USG

e. MRI foot

A girl just started her sexual life …asking for your advice.

a.do chlamydia screening 6 monthly

b.do all STI screening every year

c.cervical screening start from 25 years of age

Lady will join permanentnight shift.take balanced diet,mildly fair skinned,participate


in outdoor games.come regarding her concern abt vit D as she will nothave enough day
light exposure due to her job.what will be ur course of action regarding this lady?

a.measure her vit D status after 6 months

b.add vit D rich food with her balanced diet

c.give her vitD supplement

d.Give soralin

e.REASSURE her diet will be enough for her…

AfraTafreeh.com
Get more from: MplusX Qbank (https://member.mplusx.com/)
Facebook Page: (https://www.facebook.com/mplusxqbank/)

old lady with painful swelling infront of tragus with redness.Cause?

a. duct stenosis
b. duct stone
c. coxakie virus
d. poor oral hygiene
e. parotid duct Ca
https://www.ncbi.nlm.nih.gov/pmc/articles/PMC2740856/

32yr woman with no hx of cancer or any illnesses, no f/h of cancer, comes for screening

a.3yearly fobt

b.2yearly colonoscopy

c.5yearly mammography

d.3 yearly pap smear

e. 2 yearly fobt

You are a junior doctor. A 16 year old girl comes with abdominal pain. She does not
allow you to touch her abdomen. You call Surgery consultant who criticize you and tell
you to call the gynaecology consultant. What to do?

a.Re-examine patient

b. Call Gynaecology consultant

c. Do pregnancy test

55yr male,smoker,h/o surgery for ca oesophagus, swelling in the lateral neck 3


weeks,non tender,smooth,soft,free from overlying skin.oropharyngealexamination
normal.whts Dx?
a.branchial cyst
b.pleomorphic adenoma of parotid

AfraTafreeh.com
Get more from: MplusX Qbank (https://member.mplusx.com/)
Facebook Page: (https://www.facebook.com/mplusxqbank/)

c.submandibular duct stone


d.metastasis from cancer

An old man with onset pain in the lower back was given indomethacin. With little
weight loss, anal sphincter tone was normal. On DRE, prostate was bit enlarged and
smooth. Patient now complains of hematuria. What is dx?

a. Indomethacin induced interstitial nephritis


b. testicular cancer
c. abdominal cancer
d. acute pylonephritis
e. kidney failure

75 years of age has 3 week history of progressive constipation.history of appendectomy


and cholecystectomy. Xray mentioned (not given) dilatation of small intestines. Which of
the following is the most likely diagnosis?

a. Sigmoid volvulus
b. Caecal volvulus
c. ca caecum
d. adhesive IO

25 year old on OCP with post coital bleeding, last pap 18 months ago was normal. what
next

a.Assure

b.Colposcopy

c. Repeat Pap

d. Do Thin Prep

AfraTafreeh.com
Get more from: MplusX Qbank (https://member.mplusx.com/)
Facebook Page: (https://www.facebook.com/mplusxqbank/)

e. Check for HPV

1.Intoxicated child scenario ..took sips of drink from everyones drink at the party.parents brought
the child in er with stupor, lethargy and unconscious,next initial step asked?

1.b glucose

2.b alcohol level

3.SE

4.EKG

5.Urine drug screen

https://kidshealth.org/en/parents/hypoglycemia.html

2.vit c given to one group of patients whereas other group was control group –to study the effect of
vit C in upper resp tract infection.what to do to know the validity of the study?

1.randomization

2.random up

3.

3.One picture with ? blood stained or erythematous vesicles in clusters present along t 10
dermatome around umbilicus with on eblackish spot in the lesion.

40 y /lady admitted to hospital for severe upper abdominal pain 4 days ago.No similar previos
history mentioned.what is the next management?(pic more severe than this arund umbilicus)

1.oral acycloviri/v acyclovir

2.continued analgesia

3.i/v hydrocortisone

4.oral prednisolone

4. Another skin lesion on forearm of a child honey crusted in appearance – vesiculopapular tyoe
around 5 cm in size rounded,2 in number surrounding skin normal.asked for school exclusion?

1..2 days after commencement of antibiotic

AfraTafreeh.com
Get more from: MplusX Qbank (https://member.mplusx.com/)
Facebook Page: (https://www.facebook.com/mplusxqbank/)

2.no school exclusion nedded

3.7 days after commencement of antibiotic

5.37 wks primigravida admitted with right abdominal pain.o/e bp: 130/80,pr 80/min,temp 37.8,fhs
160/min,on investigation tlc 21*10__9.next step in management asked?

1.iol

2.i/v antibiotics

3.caesarian section

https://www.ncbi.nlm.nih.gov/pmc/articles/PMC5830160/

56y/f h/o hypertension,t2 dm controlled presented with sudden loss of vision with 24 hours on
examination rt eye 6/36 left eye 6/6. on fundoscopy pic no macula was given ,only optic disc pale
retinal vessels, there was a clot visible inside one of retinal artery. red reflex also present. asking
cause?

1.crvo

2.crao

3.diabetic retinopathy

4.grade 3 hypertensive retinopathy

5.cataract

7.ct picture given.45y/male presented with 4 month h/o weight loss of 5 kg,malaise ,nausea ,dry
rentching….. I excluded other option by ct. similar to this transvere section

AfraTafreeh.com
Get more from: MplusX Qbank (https://member.mplusx.com/)
Facebook Page: (https://www.facebook.com/mplusxqbank/)

was a case of gastric outlet obstruction

8.2 xray on sbo..one asked diagnosis and other asked next immediate mangement (nasogastric tube
decompreesion)

9.one case of atrial flutter 2:1 ,lots of p wave asked management.scenario was 70 y old man
experienced lightheadedness and dizziness of short duration presented in ed.o/e bp 80/40??(ecg
given)

1.defibrillation

2. dc version

3.metoprolol

4.1/v adrenaline

https://www.msdmanuals.com/en-au/professional/cardiovascular-disorders/arrhythmias-
and-conduction-disorders/atrial-flutter

AfraTafreeh.com
Get more from: MplusX Qbank (https://member.mplusx.com/)
Facebook Page: (https://www.facebook.com/mplusxqbank/)

10.another ecg picture not given,but scenario of stemi was described (st segment changes) with few
hours history of tightness in the chest and severe pain with sweating for 12 hours. next step in
treatment asked?

1. thrombolytic therapy

2.i/v glyceryl trinitrate infusion +aspirin

3.morphine

4. pca

11.famous mother and daughter recall_ mother asking about reports of daughter

12.one question on juvenile pyromania_ ans curiosity toward fire(find the scenario in amedex)

13..fall question ..lady 80y living alone with /o t2dm,htn taking multiple drugs..asked cause of fall?

1.living alone

2.polypharmacy

3.drug interaction

14. 92 year old female,admitted in hospital for pneumonia,she wants her DNR order to be
documented in her treatment file.which of the following is to be assessed

before documenting her wish?

1.competency

2.mmse

3.ask her family members about her decision

4.advocate

5.ask her previous physician

15.2 year old child..only babbles other developmental milestones normal.Asked cause?plays
peekaboo with friends,sits without support at 8 months and stands with support at 12 months..

1.normal variant

2.speech delay

16.spinal muscular dystrophy scenario (decreased power of all limbs but normal reflexes..recurrent
upper and lower resp tract infection)

AfraTafreeh.com
Get more from: MplusX Qbank (https://member.mplusx.com/)
Facebook Page: (https://www.facebook.com/mplusxqbank/)

17.25y old man presented in ed with h/o rash and fever within 24 hours.o/e lethargic,drowsy neck
stiffness present.which one of the following is the appropriate step to reach diagnosis?

1.blood culture

2.chest xray

3. lumber puncture

18.h/o rash after penicillin injection..asked what next antibiotic?

1,cephalaxin

2.ciprofloxacin

3.roxithrtomycin

4.ticarcillin

https://www.ncbi.nlm.nih.gov/pubmed/16451776

19.25y old man presented in ed with h/o rash and fever within 24 hours.o/e lethargic,drowsy neck
stiffness present.which one of the following is the most appropriate management?

1.intrvenous cephalaxin

2.intravenous ciprofloxacin

3.oral roxithromycin

4. intravenous ticarcillin

5.intravenous penicillin

20.new recall.gloves and stocking paraesthesia,difficuilty in walking upstairs,decreased tone in both


the lower limbs,reflexes are increased in 55y/m of one month duration.there is no prodrome of
urti.asked investigation to reach diagnosis?

1.mri spine

2.ct head

3.lumber puncture

4.nerve contusion studies(scenario was confusing could not reach to diagnosis)

https://www.racgp.org.au/download/documents/AFP/2011/December/201112tsang.pdf

AfraTafreeh.com
Get more from: MplusX Qbank (https://member.mplusx.com/)
Facebook Page: (https://www.facebook.com/mplusxqbank/)

21. OSA question—asking the most appropriate management in 38y/ male bmi 34 with increased
daytime hypersolmenance,had to quit job because of his hypersolmonence at work,wife is worried
and asking what is the best treatment for him?

1.aggresive wt reduction and dietary modification best

2.sleep hygience

3.cpap and sleep conduction studies next

22.A Young man presented with pain lower abdomen and bloody diarrhoea with mucous for 2 week
swith wt loss.he had similar episode in the past.just came back from Somalia..what is the likely
diagnosis?

1.amoebiasis

2.giardiasis

3.infective colitis

4.ulcerative colitis.

https://emedicine.medscape.com/article/212029-clinical

23.which one of the following is the most consistent finding with bleeding per rectum due to
haemorroids?

1.painful defecation

2.mucus in stool

3.loosenng of stool

4.foul smelling stool

https://www.ncbi.nlm.nih.gov/books/NBK279467/

25.a 20y/f presented in ed with pain upper abdomen ,mildly icteric with history of loss of appetite
for 2 weeks ,nausea and vomiting ,with general malaise,yesterday she took 10 tablets of
paracetamol for her headache in 2 to 3 hours interval.

no examation findings(im pretty sure)on investigation:

tbil: 50

alt:1180

AfraTafreeh.com
Get more from: MplusX Qbank (https://member.mplusx.com/)
Facebook Page: (https://www.facebook.com/mplusxqbank/)

ggt: raised

alp:normal

paracetamol level: noramal

asking the cause of her presentation?

1. alcoholic hepatitis

2.paraetamol poisoning

3.viral hepatitis

26.intern sleeping in emergency during her night shifts.whom shoul you inform?

1.emergency director

2.her emergency superisor register

3.aphra

4.hospital director

27 pantoprazole ethic question,another ethic question: note discrepancy between the lady and
pharmacist question.

28.8 year old boy presnts with dysphaga to both solid and liquid foods for 1 month,his brother is
aknown case of asthma well contolled by salbutamol.o/e chest clear..what is the next investigation
to reavh diagnosis?

1,ct scan thorax

2.spirometry

chet xray

4.barium swallow

29.another recall mother doesn’t wnt to give vaccine to her child,no previous immunization history
present..what to do next?

1.refer her to community nurse.

2.involve social health worker

3.refer her to another doctor

4.respect her wish

AfraTafreeh.com
Get more from: MplusX Qbank (https://member.mplusx.com/)
Facebook Page: (https://www.facebook.com/mplusxqbank/)

30. post operative day 4 presented with confusion.on evaluation:

urine osmolality low

serum osmolality low

na: 123

k:3.5

other parameter normal.what is your next management?

1.1.8% hypertonic saline infusion in 1 hour

2.0.9% ns in 24 hours

3.restrict fluid intake for next 24 hours

(there was no option for 3% hypertonic saline)

31.middle aged women with pain in right thigh woese at night and morning.but gets better after 10
minutes of walk.history of bilateral knee osteoarthritis. next investigation?

1.xray right hip

2.bone scan

3.xray lumbar spine

4.mri

(Looks like vascular rest pain..maybe options missing)

31.that famous anterior inferior dislocation of shoulder .xray given.(head ofhumerus in ant chest
wall ).asked which one of the following will be consistent on examination?

1.loss of sensation over medial aspect of arm

AfraTafreeh.com
Get more from: MplusX Qbank (https://member.mplusx.com/)
Facebook Page: (https://www.facebook.com/mplusxqbank/)

2.loss of sensation over medial aspect of forearm

3.loss of sensation over lateral aspect of shoulder

4.tingling sensation over the hand

32. ovarian metastasis on ct …in 28y female ,,asked most common primary?

1.breast

2.lung

3.colon

4.stomach

5.liver

33.case of status epilepticus in a lady 4 weeks postpartum(no previous history of seizures)

treatment asked after giving i/v midazolam in ed?

1.phenobarb

2.phenytion

3.Mg Sulphate

4.carbamazepine

5.daizepam

34.2 recalls on check serum antibody.

35.2 recall on pop indications

37. preeclampsia lady ,asking which on of the following is the important indication for urgent
admission?

1.ankle clonus 10 times

2.hand edema

3.increase in bp 180//100

36. 30 year man with difficulty initiating urine n dribbling at end he says he empties bladder completely he
doesn’t has to wake up at night to void urine cause

a.BPH
b.Ca prostate

AfraTafreeh.com
Get more from: MplusX Qbank (https://member.mplusx.com/)
Facebook Page: (https://www.facebook.com/mplusxqbank/)

c.urethral stricture
d.ca bladd

37.another recall hepatitis two doses given at 2 months and 4 months of gae ,but did not get other
vaccine due to pneumonia,now presents to you at 8 month f age with mild fever and cough .o/e
everthing normal.what to do regarding vaccination?

1.restart immunization schedule

2.give hep b vaccine now and recommence follow up schedule

3..give the last dose after 2 months

38.cxr..given cardiomegaly,straight right border of the heart,increased pulmonary vascular


markings.28y old man went to Somalia to work for 2 years and returned back.Now presents to you
with sore throat,fever and cough for one week.on examination mild cervical
lymphadenopathy,pansytolic murmur at apex with bibasal crepts.what is the likely diagnosis?

1.pulmonary tuberculosis

2..pulmonary infarction

3.rheumatic heart disease

2 more option for pulmonary pathology

39.acute ototis media child scenario..causative organism asked?

1.step pneumonia

2.step mycoplasma

3.staph aureus

4.haemophilius influenza

40.old lady presented to e/r 4 hours after mva with severe retrosternal chest pain after accident.o/e
bp: 180/90mmhg on both arms,pulse 80/min,temp :afebrile.cxr was given.(clear widening of
mediastinum) asking diagnosis?picture similar to this

AfraTafreeh.com
Get more from: MplusX Qbank (https://member.mplusx.com/)
Facebook Page: (https://www.facebook.com/mplusxqbank/)

1.thoratic aortic dissection

2.myocardial infarction

3.esophgeal rapture

4.pulmonary embolism

41.3 weeks history of recurrent pain in the epigstrium in 45y/m lawyer,no known aggrevating or
relieving factors.o/e mild epigastric tenderness,otherwise normal.

asking diagnosis?

1.esophageal stricture

2.esophageal ca

3.esophagitis

4.duodenal ulcer

42.one lobar pneumonia scenario in 58y male with some underlying condition.o/e temp 38,bp
130/80,pr 100min. cxr picture given.asking treatment?

1.i/v penicillin

2.oral roxithromycin

3.amoxiclav

AfraTafreeh.com
Get more from: MplusX Qbank (https://member.mplusx.com/)
Facebook Page: (https://www.facebook.com/mplusxqbank/)

4.cephalaxin

1.A guy came back from Angola, high fever, you check for malaria, took antimalarics, twice negative
results for malaria in 24 hours apart, the test has 98% sensitivity, 96% specificity, what to do next?
a)Check for different disease
b)Repeat blood smear-thick and thin after 24 hours
c)test in high peak of fever

2. Patient always keeping isolated himself from others

a) Loose association

Rest are forget

4. Man comes to your clinic with chronic schizophrenia. He is well controlled on drug. Before
entering your office,he stops and salutes you three times. This is an example of??
a) a mannerism
b) dystonic reaction due to antipsychotic drug
c) tourette syndrome.

5. One neuro question patient rt sided facial burning sensation,ataxia,nystagmus,pain and temp loss
in rt side and left sided weakness,reflexes are exacerbated (side forgot) askes dx.

a.Right tempero occipital infarct


b.Right lateral medullary syndrome
c.Right pontine syndrome

7.

Similar ECG (Hyperkalemia), patient taking multiple drugs, diagnosis asked :

a.Hyperkalemia

b.Hypokalemia

8. ECG (1st/2nd degree heart block).

AfraTafreeh.com
Get more from: MplusX Qbank (https://member.mplusx.com/)
Facebook Page: (https://www.facebook.com/mplusxqbank/)

A 65 years old Pt presented by light headiness, and palpitation, Examination was clinically free, he’s
on perindopril, Eltroxin(thyroxin), verapamil, antidiabetic medications. What’s the next appropriate
treatment?

A. Increase Eltroxin
B. Stop verapamil
C. Stop perindopril
D. Add digoxin

9.Patient presents with sudden severe chest pain ,sweating and vomiting.Physical examination
normal. All lab findings including troponin are normal and ecg also normal..Next management :

a.Repeat troponin

b.Repeat ECG after _ hourb

10. CT scan of diverticulitis in elderly woman with scenario of fever, LIF pain, mild leukocytosis,
history of constipation. Most appropriate next treatment?(CT given)
A. Broad spectrum antibiotics with gut rest
B. Prednisolone
C. CT guided drainage
D. Laparoscopic surgery
E. sigmoid resection

11. Ankylosing spondylitis x-ray. Paracetamol, codeine taken but not improved
a. Indomethacin (no naproxen in option)
b. prednisolone
c. Methotrexate
d. Infliximab

13. HSP scenario with leg photo, develop rash in buttock and leg abdominal pain, next investigation
A) abdominal USG
B) urinary phase-contrast microscopy
C) blood culture
D) CXR

14. Picture of facial palsy 1 day and ear discharge for 2 days,. asking treatment?
Steroid
Famcyclovir

15. similar picture

AfraTafreeh.com
Get more from: MplusX Qbank (https://member.mplusx.com/)
Facebook Page: (https://www.facebook.com/mplusxqbank/)

Man treated with cefalaxin for sore throat 6 days later presents with rash
A. Delayed type 4 hypersensitivity
B.SJS
C.TSS

16. Similar picture

Swelling in the anterior neck similar to this picture but a little smaller
Asking what will you find on physical examination
A palpable hard nodule under tongue
B movement on swallowing
C movement on tongue protrusion

17. 17 yrs boy says he must count till 20 other wise her mom will be killed. They has a minor
accident 3 months ago. Boys keeping him self in the room most of the time. He is hearing the
voices but could not recognize it. What is Diagnosis

A. OCD
B. Major Depression
C. Schizophreniform Disorder
D. PTSD

18. Drawing pentagon, which hemisphere test..with scenario

a. Fronto parietal
b. B.dominat parietal
c. Occipital
d. D.temporal
e. Non domi parietal

https://www.redalyc.org/jatsRepo/4835/483551472009/html/index.html

19. Woman has regular menses from 4-6 wk interval , pain on left iliac fossa for 3 days,
aggravated while urination, her lmp was 9 days back , she had similar episode 4 wks back
which lasted for 3 days , cause ?
a. Ovulation
b. complicated ovarian cyst

AfraTafreeh.com
Get more from: MplusX Qbank (https://member.mplusx.com/)
Facebook Page: (https://www.facebook.com/mplusxqbank/)

c. dysmenorrhea
d. early PID

Ans: Endometriosis

20. 22 weeks pregnant comes with c/o vomiting and pain in lower abdomen . bp 130/80
pulse 85 min, no fever . there is tenderness in pelvic region . fetal heart rate 140 beats / min.
what is cause of pain?
A. Torsion of corpus luteum
B. Round ligament pain
C. Complex ovarian cyst
D. Ectopic pregnancy
E. red degeneration of unrecognized fibroids

21. Man with scenario of claudication in left calf. On exam, all peripheral pulses palpable on right
side. Left side all pulses palpable but dorsalis pedis weak and hardly palpable. Bilateral
weakness of ankle reflex and loss of sensation in foot. side ABI on right side 1, left 0.75.
Appropriate investigation?
A. Conventional angiogram
B. CT angiogram
C. CT spine
D. EMG
E. Arterial Doppler

22.A 7 years old girl with soiling in her underwear, she tells lies about accidents at school and hide
her soiled underpants and cry and quarrels with her brother when he calls her smelly. What is your
diagnosis?

a)Regression

b)Conduct disorder

c)Oppositional Defiant Disorder

d)Delayed milestones

e)Depression

23.Child fell off motorcycle, she wasn’t wearing helmet, vomitted twice. GCS 15 when examined and
she was playing on her ipad in the observation area. slight ?Left forehead bump +. No other
significant findings. Her father wants the child to go back home with him before observation period
is over, despite counselling him the contrary. What will you do??

ask the child if she would like to go back or not

explain that the child needs to stay here until it’s time for safe discharge

Sign DAMA and let her go

24.Woman fall on outstretched hand at the back door of her house. Her xray on 6 weeks
postfracture heal finely. She came to followup on 8 week post fracture only show mild intermittent
pain. Mx?

a) Repeat xray wrist

AfraTafreeh.com
Get more from: MplusX Qbank (https://member.mplusx.com/)
Facebook Page: (https://www.facebook.com/mplusxqbank/)

b) Sling for better stability

c) CT

d) Arrange occupational therapy for back door safety

25. A 29-year-old woman has been treated with sertraline 50mg daily for the past five weeks. She
had initially presented with a four week history of bouts of crying, insomnia, tiredness and weight
loss as well as a general loss of interest in her family, friends and her job. Her symptoms had
followed the break up of a longterm relationship and have improved with the treatment. She is now
complaining of persistent abdominal discomfort over the past month. Which one of the following is
the best initial management strategy at this stage?

A. Switch to another selective serotonin reuptake inhibitor (SSRI)

antidepressant.

B. Switch to a selective noradrenaline reuptake inhibitor (SNRI)

antidepressant.

C. Switch to a tricyclic antidepressant.

D. Stop the sertraline immediately

E. Augment the sertraline with a mood stabilizer.

26.Family with a newborn baby moved to a small village in rural Australia, father called up GP to
inform wife sad all the time, low mood, crying but taking good care of baby, cannot get her to the
hospital as he is busy with new work, what next:

a-Make a home visit

b-Send a social worker to check on woman

c-Insist husband to get the woman & baby to clinic

d-Admit the woman, baby can be with dad

e-Wait for the husband to call back again

27.man his father is dementia or Alzheimer ..he cannot take care of him anymore

A social work review

B immediately arrange a nurse to home caere

C immediate arrange admission

D prescribe drug

28. 35 year old womenHad gestational DM . What will u suggest for screening?

Fbs 1 yearly

Fbs 3 yearly

AfraTafreeh.com
Get more from: MplusX Qbank (https://member.mplusx.com/)
Facebook Page: (https://www.facebook.com/mplusxqbank/)

HbA1c at the 1st TM in next pregnancy

https://www.racgp.org.au/clinical-resources/clinical-guidelines/key-racgp-guidelines/view-all-racgp-
guidelines/management-of-type-2-diabetes/reproductive-health/gestational-diabetes

29.old lady with htn,dm on lots of drugs.now came with increased drowsiness and loss of appetite.

Given lots of labs Cbc,rfts,tfts,lfts,Ca phosphorus level.

Everything was normal except urea and creatinine it was high

a.iv normal saline

b.hemodialysis

c.ca gluconate

d.resonium

https://www.aafp.org/afp/2005/1101/p1739.html

30. 1 year old child diagnosed with bacterial meningitis was admitted to hospital.blood culture taken
and iv ceftriaxone started.after 1 day of admission child had a seizure of <1 min duration.long labs
were given sodium and bicarbonate low.all others were in normal range.what is the cause of
seizure?
a.SIADH
b.dehydration
c.adrenal failure

https://emedicine.medscape.com/article/126806-overview

31. post operative (cancer colon) with good hydration with long lab..
Na-decrease
S.Osmolarity-increase

U.osmolarity-decrease
1-hypertonic saline
2- normal saline
3-restrict fluid to 1 and half litter
32. child 10 yr old refuses to go to school . he developed frequent touching of his face and raise his
arm above his head and crawl his hand on the ground, he can supreess doing those thing s but
developed anxiety if he suppress those things . his classmates make fun of him coz of those

AfraTafreeh.com
Get more from: MplusX Qbank (https://member.mplusx.com/)
Facebook Page: (https://www.facebook.com/mplusxqbank/)

behaviours and his teacher also annoyed,what is the diagnosis?


a) asperger disease
b) ADHD
c) complex motor tic disease
d) Tourette syndrome

33.Case of old woman with iron deficiency anemia, no symptoms no complain, cause?
A. Carcinoma of caecum
B. Carcinoma of.....
C. Carcinoma of stomach
D. nutritional deficiency

34.Old px underwent surgery of femur fracture, 6 days after had dyspnea and confusion. Temp-
37.6,On auscultation, there is crepitation in right lower lobe.
A. PE
B. fat embolism
C. pneumonia
D. aspiration syndrome

35. 3 yr old from Bangladesh, fever cough for 1 month. Xray already done..asking next inv
a. mantoux test
others don't remember

36. MS scenario(optic neuritis)..asked Ix :

a.MRI

b.VEP

37. Woman 55 years came to doctor for regular medical check up. Her mother has CA cervix at 63
years. Maternal aunt has breast CA at --- year . and father has colon cancer at 72 years. What to check
her?

a. Colposcopy 2 yrly
b. 5 yrly cervical cancer screening
c. 5 yearly colon cancer screening
d. mammogram 2 yearly

38.Breast CA screening scenario. Pt BRCA 1. What else you will do?


A. Vaginal USG 12 monthly
B. Bimanual examination 6 monthly
C. CT 6 monthly
D. Cervical screening 12 monthly.

https://www.cancer.gov/about-cancer/causes-prevention/genetics/brca-fact-sheet

39.patient presents with fever,vomiting ,diarrhea for 2days..on examination jaundice,tender


hepatomegaly 5cm..increase S. bilirubin level and ALL the liver enzymes are elevated .. Dx asked :

a.Hep A

AfraTafreeh.com
Get more from: MplusX Qbank (https://member.mplusx.com/)
Facebook Page: (https://www.facebook.com/mplusxqbank/)

b.Hep B

c.Hep C

40.A patient with prostate ca T3b N0M0. Mx


A.Radical prostactemy
B.orchidectomy
C. External beam radiation therapy
D. Androgen deprivation

41. 74 years old man is having a check up for prostate cancer last two years PSA was 1.5mg/ml, now
PSA is 3.8mg/ml, then 12 slides of biopsy done and only one foci showed adenocarcinoma. Gleason
score is 4. How will you manage this patient?
A.TURP
B.radical prostatectomy
C.active surveillance
D.EBRT
E. Androgen therapy

42. Bronchiolitis senerio of 2 years old boy..asked how to confirm the dx :

a.Chest Xray

b.Improved with salbutamol

43.Woman with h/o steroid use & previous fracture, taking Risedronate 35mg, has
recurrent vertebral fracture in last 3 months, what next:
a.stop risedronate and change to IV zolendronic acid
b.Change to alendronate
c.Continue Risedronate
d.Stop Risedronate

https://www.ncbi.nlm.nih.gov/pmc/articles/PMC3383538/

https://www.osteoporosis.org.au/sites/default/files/files/RACGP%20Osteoporosis%20Summary%20
Guideline%204428%20Nov%202017.pdf

44. Human bite recall. 35 years old man went to pub, where a woman bite him, totally unknown and
she went away. 2 cm lesion, bleeding continues, teeth mark, he is fully tetanus immunized 3 years
ago. What to do now?
A-suture and review in 24 hours
B-hep B vaccine and immunoglobulin
C- zidovudin
D- Azithromycin
E-tetanus immun

(No option for flucoxacilin)

45. non healing diabetic ulcer duration not remembered ,both dorsalis pedis pulsse are absent. Xray
done showing osteomylitis..asked most appropriate investigation?

AfraTafreeh.com
Get more from: MplusX Qbank (https://member.mplusx.com/)
Facebook Page: (https://www.facebook.com/mplusxqbank/)

a. MRI
b. Arterial droppler study

46. 37 wk pregnant lady with mild pre-eclampsia for induction of labour. Bishop score 2.What will
you do?

Elective LUCS

Induction with PGE 1 and ARM after 4 hours

Induction with PGE 2 and ARM after 12-24 hours.

Urgent LUCS

Syntocinon infusion

47. Middle age man present with retrosternal chest pain. This is preceded by
vomiting. On examination, reduced breath sounds in lower lobes of lungs
and dullness over these.Most appropriate investigation?
a. CXR
b. Gastrograffin swallow
c. CTPA
d. Non contrast CT chest
e. ECG

48. Man with sharp pain radiating to chest,,, also present with some neurological c/F ..knee
jerk increase,,ankle reflex increase,,weekness in both lower limb ,,no other abnormality,, no
skin lesion,,, what most appropriate next step--

Chest X-ray
Ct spine
Varicella serology
Bone scan
Plasma electrophoresis

https://www.ncbi.nlm.nih.gov/pmc/articles/PMC3139357/

49. Restless leg syndrome scenario. What is the next investigation?


a) Sleep studies
b) Nerve conduction test
c) Iron studies

50. Patient after MVA ,laboured breathing,cxr shows no pneumonia, trachea centrally place.asked
next??
A. Intubation and IPPV
B. Iv fluid
C. Chaest drain
D. Needle thoracocentesis

AfraTafreeh.com
Get more from: MplusX Qbank (https://member.mplusx.com/)
Facebook Page: (https://www.facebook.com/mplusxqbank/)

CXR required

52. A women on multiple drugs one of them was digoxin, she presented with central abdominal pain
and tenderness. Her pulse was irregular. what is appropriate next investigation?
A. Abdomen USG
B. CT abdomen
C. Serum lipase
D. Digoxin level
E. INR

No option for serum lactate

53. Lady h/o hysterectomy 15 yr back c/o abdominal distention and severe pain ,vomiting .after
some times relieved..P/A : mild tenderness .. diagnosis :
sigmoid volovus
close loop obstruction without ischemia
bowel ischemia

54. Similar picture

Waldenstrom macroglobulinemia (pic)


patient came with rigors and fever and right lower lobar pneumonia or some infection like this
admitted and started ampicillin and azithromycin minimal time generalised vesicular rash appeared
asking management
a. Immunoglobulin
b. cease ampicillin and start ceftriaxone
c. prednisolone
d. Ganciclovir
e. Amphotericin

https://www.ncbi.nlm.nih.gov/pmc/articles/PMC3133652/

56. pt with hx of injury to eye. 2 days later present with decrease visual acquity anterior chamber is
clear. dx?
a) glaucoma
b) vitrous haemorrhage
c) detached retina
d)cataract

AfraTafreeh.com
Get more from: MplusX Qbank (https://member.mplusx.com/)
Facebook Page: (https://www.facebook.com/mplusxqbank/)

57. 89 year old woman in in nursing care facility. Nurses complain she maturbates in private. What
next.
1.Shift here to 4 bed room ward
2.Talk to her fam about it
3.Talk to the resident that its inappropriate
4.Tell nurses to give her privacy
5.Neuro or psych consultation

58. you take a picture of pt's pilonidal sinus for assessment with agreement of pt. when you can use
it?
1. de-identification picture can be used for medical journal
2. save in PERSONAL phone and check it for pt's disease progression
3. it can be shared in SECURE internet with colleagues for treatment

59. An old woman came to the ED with vulval erythema and vaginal pain. She had some symptoms
of candidiasis which was confirmed by labs. How will you treat this woman?
a.Topical Nystatin
b. Oral Fluconazole
c. HRT
d. Topical estrogen
e. Oral Clotrimazole

60. Pt admitted to hosp due to gentourianry infection (not sure). History of rash to amoxicillin. staph
aureus on culture which is the antibiotic of choice for her
A.Ciprofloxacin
B.Cefazoline
C.Cefalaxine
D.vancomycin
E.ceftriaxone

61. Pt had melanoma on face 2.5 mm. Excised with 2.6 mm margin. What to do now.
A. Follow up after 3 month
B. Wide excision again
C. Chemotherapy

62. patient wants to be your friend on social media what you should do?
A. Tell them they can discuss only medical related issues
B. They can message you directly
C. Accept the request but don’t contact directly
D. Decline the request
E. Delete your social media site

63. 16 years old girl came to you besides contraception and safe sex what advice you would like to
give her?
A. Check chlamydia 6 monthly
B. Do all STDs 12 monthly
C. Cervical cancer screening from age 25years
D. Advise her to take HPV booster in 5 year

64. Abortion at 10 wks , no other features…cause asked :

AfraTafreeh.com
Get more from: MplusX Qbank (https://member.mplusx.com/)
Facebook Page: (https://www.facebook.com/mplusxqbank/)

a. Chromosomal abnormalities

b. Cervical incompetency

68. Young male patient with h/o childhood used asthma, has trouble swallowing certain types of
food, Endoscopy showed strictures, biopsy eosinophilia & IgA, what next:
Fluticasone
PPI
Antibiotics
Surgery

72. 9 month old baby brought in by parents worried of head size... child born normal with no
medical problem. Growth at 25-50th centile. Examination had flat occipital and left prominent
forehead. what to do?
Change sleeping position
CT
Ultrasound
MRI
Resaaure

73. Old man 75 years of age has 3 week history of constipation. He presents with mild abdominal
tenderness and on rectal examination there is no faeces in the rectum. On CT there is cut off at the
sigmoid colon and dilation of the ascending, transverse and descending colon. Which of the
following is the most likely diagnosis? (no ct given)

a. Sigmoid volvulus

b. Colon cancer

c. Caecal volvulus

75. 13 yr old boy, stealing , swearing, destructing furniture, violent towards teachers and classmates,
diagnosis
A- ODD
B- conduct disorder
C- borderline in adolescence
D-ADHD

76. An old woman comes to you for foul smelling and malodorous watery vaginal discharge. She is in
sexual relationship only with her husband. What can be the cause?
Bacterial vaginosis
Trichomonous
Chlamydia

79. One night you see your patient driving his car , when he is not allowed to drive at night. And you
are sure that it was your patient, not anyone else. What will be your most appropriate step?
a) Inform to driving license authority without knowledge of patient
b) Invite patient to discuss with him first
c) Ignore

81. GBS scenario asked about,Respiratory Monitoring.....


FVC

AfraTafreeh.com
Get more from: MplusX Qbank (https://member.mplusx.com/)
Facebook Page: (https://www.facebook.com/mplusxqbank/)

PEFR
Pulse oximetry

82. Mother primi presents at 35weeks with breech presentation


Plan of management?
ECV now
ECV after two weeks
LSCS
Wait for spontaneous onset of labor

83. 52years old around menopause, taking HRT for 2 years..no other features.. what risk :

a. Stroke

b. VT

c. Breast cancer

84. Man 35 yrs with h/o difficulty in micturation for 2 yrs. Difficult to start and dribbling at
the end of micturation. But no complain of frequency and urgency.Dx :

a. BPH
b.Bladder stone
c. Urethral stricture
d.Cancer

85. Old woman with urgency and nocturia for a long time. Now having incontinence for like a month.
She couldn't reach the toilet downstairs. She also has bilateral knee OA. What is the best
management for her?
A. Fluid restriction at night
B. Install a commode in her bedroom.
C. Oxybutynin
D. Paracetamol

87. Sudden severe headache, no fever,CT normal…Dx :

a. SAH

b. SDH

c. EDH

88. 45 year old patient with previous history of DVT now undergoing colectomy surgery.Which is
best for him?
a.Heparin for 10 days post op
b.Give heparin until discharge
c.Give for 24 days

91.30 years old woman wants to conceive 12 months of amenorrhea, bmi 30,previously irregular
menstruation 5-6 times a year. Lab values fsh 2(3.3-15), Lh 6(n),prolactin, estrogen, tsh normal.
What will help in Dx?

A.s.progesteron

AfraTafreeh.com
Get more from: MplusX Qbank (https://member.mplusx.com/)
Facebook Page: (https://www.facebook.com/mplusxqbank/)

B.s.testosterone

C.MRI pituitary

D.Pelvic usg

92. pt of crohns disese was on sulfasalzine prednisonlone,azathioprine labs given neutrophils and
wbc low, hb low, platelate low .. cause :
Sulfasalazine

predniosonlne

azathioprine

93. What you will screen before introducing infliximab in a patient with Crohn disease?
a. QuantiFERON-TB Gold test
b. FBC
c. LFT
d. ECG
e. Urine cytology

94. Middle age man present with retrosternal chest pain. This is preceded by vomiting. On
examination, reduced breath sounds in lower lobes of lungs and dullness over these. Next
appropriate investigation?
a. CXR
b. Gastrograffin swallow
c. CTPA
d. Non contrast CT chest
e. ECG
95.A 5 day old baby born at term suddenly collapse and was severely dyspnoeic while still in the hospital.
Examination reveals a centrally cyanosed baby, tachycardic HR-164 b/m and tachypneic RR %56cpm,
temperature was normal, spO2 78%. No murmur heard on cardiac auscultation. What is your diagnosis?

a. PDA

b. Transposition of great vessels with associated VSD

c. Hypoplastic left heart

d. Patent foramen ovale

e. Tetralogy of Fallot

96. Old patient. Gradual LOV for 5 months which improved with pin hole
a. Cataract
b. Diabetic retinopathy
c. Hypertensive retinopathy
d. ARMD

No option for refractive error

98. Mother brings her 10 month old boy, who is exclusively breast feeded first 6 months no problem
that time..then started weaning, for the last 2 month he developed diarrhea , pallor,failure to
thrive.. Ix asked :

AfraTafreeh.com
Get more from: MplusX Qbank (https://member.mplusx.com/)
Facebook Page: (https://www.facebook.com/mplusxqbank/)

A. Antigliadin antibodies

B. Hb electrophoresis

C. Iron study
D. Serum electrolytes

99. PCOS scenario.. definitive dx :

a. FSH/LH ratio altered from 3 to 1

b. Elevated S. Androstenedione

1. A 54 year old man comes to your clinic complaining of acutely painful defecation with spotting of
blood on toilet paper. Which of the following is next appropriate management?

A.Spinctorectomy
B.Topical glycerine trinitrate.

c.Excision
D.Sclerosant Injection

E. High fibre diet

• A lady with previous pap showing LSIL. Now comes for repeat pap after one year. Her pap is
done now. What will u council her about vaccine?
a. Vaccinate now
B. Ask her the decision for vaccine will be made after today’s pap result.
c. Do a HPV serology

• 25 year old on OCP with post coital bleeding, last pap 18 months ago was normal. what next
A. Assure

B. Colposcopy
C. Repeat Pap

D. Do Thin Prep

E. Check for HP

• Lady on venlafaxine overdose after separation with her husband. She is depressed or
down?(dont remember the exact word). One year ago she was prescribed venlafaxine(not sure
what for). What will you do next? (this q was a little different from the usual recall)
a. increase the dose of venlafaxine

AfraTafreeh.com
Get more from: MplusX Qbank (https://member.mplusx.com/)
Facebook Page: (https://www.facebook.com/mplusxqbank/)

b. decrease the dose of venlafaxine


c. mirtazipine
d. clozapine

• an old women who is widow who seems well in past but since she became a widow and she
moves to the house and start to stay alone one of neighbour had noticed her strange behaviour. she
starts to dig some ground in front of the house and when neighbour reached near her and looked at her,
she then aroused him and get aggressive and accused him and all neighbors as imposters. After that she
became calm down and agreed to be seen at the medical clinic.Which one of the following explains her
condition?
A)thought forms
B) mood
C)orientation
D)delusion
E)memory

6. A woman c/o forgetfulness (age 58). He forgets where he keeps his things,easily gives up
task .during cognitive assessment,she becomes upset and didn’t want to do it.What to check in
mental state examination (MSE)?
a) Orientation
b) Hopelessness
c) Hallucination
d)imapired insight

7. Pt with hoarseness and right ant swelling in the neck move with swallawing with dullness
percussion on the upper part of sternum dx-
a. Multinodular goiter with retrosternal extension
b. Papillary ca
c. Follicular cyst
d. Medullary ca of thyroid

• resident while doing laparoscopy causes injury to liver surgeon comes and control but it
results in laparotomy what should be done?
a. Tell the patient he made error by registrar
b. Tell the patient it was complementary complication by registrar
c. Don’t tell as patient is recovered
d. Surgeon have to tell complete history to patient
e. Surgeon have to tell to patient in the present of second person
I got 3 similar questions..with some change of scenario n options, but all about registrar and
surgeon making mistake.

AfraTafreeh.com
Get more from: MplusX Qbank (https://member.mplusx.com/)
Facebook Page: (https://www.facebook.com/mplusxqbank/)

• 83 yrs old patient, living alone was asking help for her will. She always visit her GP and
buy her gifts. What should be her action?
A) Assess testament capacity
B) Help her with the will
C) tell her that her daughter can be beneficiaries
D) forgot

• 50 yo man Renal transplant pt with neck stiffness and meningitis sign start 9 month ago,
nothing mentioned about respiratory symptoms. no fever. on cxr there is well defined round opacity
in “middle right lung”, (no cxr was given just mention) cause?
A. aspergillus
B. TB
C. Pneumocystis Pneumonia
D. Lymphoma

• 26 year old lady came with complain of post coital bleeding thrice. Her previous pap
smear was done 3 months back which was normal. Asking about investigation.
A. Repeat pap
B. Chlamydia PCR
C. Colposcopy
D. Thick and thin smear

• A 17 years old girl came to you besides contraception and safe sex what advice you
would like to give her?
A. Check sexually transmitted diseases in 12months
B. Do pap in 6 months time
C. Do pap now
D. Advise her to take HPV booster in 5 year
E. Advise her to start cervical cancer screening from 25yrs.

• A pregnant woman comes with painful genital ulcers and swelling of vulva. What to give?
a. Iv acyclovir

b. iv prednisolone

c. oral acyclovir

• I got 2 CT brain, one infarction and the other one metastatic melanoma/glioma. Both with
scenarios.
• 1 scaphoid fracture xray, asking management.

AfraTafreeh.com
Get more from: MplusX Qbank (https://member.mplusx.com/)
Facebook Page: (https://www.facebook.com/mplusxqbank/)

• 2 ecgs. One of which was LVH, other I dont remember.

• Pt with a BMI of 36, comes with haematuria. CT given showing RCC. asked what will
increase the chance of DVT after his operation?
a. Nicotine stain of fingers
b. Atrial fibrillation
c. Bilateral varicose veins
d. BMI
e. Spider naevi on chest

• A picture of a herpes zoster rash on the


abdomen was given. Its been 4 days (weeks? Not sure) since the patient developed the rash.

AfraTafreeh.com
Get more from: MplusX Qbank (https://member.mplusx.com/)
Facebook Page: (https://www.facebook.com/mplusxqbank/)

Management:
A. Oral steroids
B. iv Acyclovir
C. Regular oral analgesia
D. Oral acyclovir

• A lady has painful red nodules on her


chin of tibia. She had had them for some time. She also has cough and shortness of breath for
several months now. Her ACE is 120 (normal <40). What is NEXT for Dx.
a. CXR
b. ANA
c. Skin biopsy.

• A boy was recovered from a beach in Northern Queensland. He was in pain and had jelly
fish tentacles stuck all over his legs. Prior to removing tentacles what will be your immediate step?
A. Pour vinegar
B. Give IV Morphine
C. Immerse legs in warm water
D. Give antivenom
E. IV fluids

• Pregnant lady 35weeks,foetus with bradycardia, emergency caesarian section done.


After delivery, baby still with Heart rate of 60bpm. What investigation on mother??
a- anti smith antibody
b- lupus anticoagulant
c- anti cardiolipin antibody.
d- anti ro-anti la antibody

• Aboriginal mamu recall.


• Immigrant man comes and tells you that in his country, secret police are after him and
they want to kill him as they are worried that I would leak the name of another person who was
involved in killing of a person. What will you check in his collateral history?
a. Immigration visa status
b. Child hood psychiatric disorder
c. School performance
d. the truth of his saying

AfraTafreeh.com
Get more from: MplusX Qbank (https://member.mplusx.com/)
Facebook Page: (https://www.facebook.com/mplusxqbank/)

• The moon is made of cheese, 'I came by bus’


which one of the following does this pt most likely have?
a. Depression
b. Dissociation
c. Depersonalization
d. Disorganized behavior
e. Delusion

• A young lady complains of abdominal pain while urinating, she had menstrual period 9
days ago. She says she also has this kind of pain 4 weeks ago and it lasted for 3 days. What is the
cause?
a. Ectopic pregnancy
b. Ovulation pain
c. Complex ovarian cyst
d. Endometrosis

• 8 year-old boy has left ear pain. O/E He has 39.1 C fever both left and right tympanic
membranes are bulging and there are suppurative tonsils. what is the diagnosis?
a. External otitis
b. Otitis media
c. Tonsillitis
d. Mastoiditis

A 25 years old woman with anger issues , relationship issues, done self harm by cutting wrist many
times. She has been hospitalized several times and also given pharmacotherapy but nothing worked.
What will be the next mode of treatment.
A. CBT
B. Dialectal therapy
C. Supportive therapy
D. ECT
E. Relationship therapy

Malignancies have been approved except:

A.CA liver & Nickel

B. Sexual promiscuity & CA cervix

C. beta chewing and oral ca


D. schistosomiasis and bladder ca

E. Ebv with lymphoma

AfraTafreeh.com
Get more from: MplusX Qbank (https://member.mplusx.com/)
Facebook Page: (https://www.facebook.com/mplusxqbank/)

A boy was recovered from a beach in Northern Queensland. He was in pain and had jelly fish
tentacles stuck all over his legs. Prior to removing tentacles what will be your immediate step?

A. Pour vinegar

B. Give IV Morphine

C. Immerse legs in warm water)

D. Give antivenom

E. IV fluids

What is the probability of the affected child for Thalassaemia Minor in case of both parents ?

0.25
0.5

0.75

100

In Australia bush fire are common either accidentally or some people lighting fire deliberately.
Which is true regarding Juvenile pyromania?

A.To conceal the crime


B.Fascinated by fire and fire equipment

C. Done for notoriety or publicity

D. Start the fire & panic when bigger

A 4days old infant found in cot cyanosed but no murmur heard, O2 saturation decreased in spite of
giving O2, asking Dx?

a) Transposition of great vessels


b) hypoplastic left heart failure
c) TOF
d) VSD
e) ASD

Man brought to the rural clinic after he was bitten by a snake through his trouser. There are two bite
marks on his leg. Which of the following is the most appropriate management?
a. Give antivenom
b. Apply torniquete and leave it for 2 hours

AfraTafreeh.com
Get more from: MplusX Qbank (https://member.mplusx.com/)
Facebook Page: (https://www.facebook.com/mplusxqbank/)

c. send the patient home as there are no signs and symptoms


d. Tell the patient that it is more likely to be a non-venomous snake bite

e. Observe the patient and give antivenom if s/s develops

Patient came back from Asia and low grade fever and cough for weeks. what is the most likely
diagnosis?

a. lymphoma

b. TB

c. Viral pneumonitis

d. pulmonary embolism

Pt with RA she takes ibuprofen and methotrexate to control her disease for 5 years. patient
complains of some symptoms, and her labs are given
protein - 9 (6-8)
albumin - 4 (3.5-5.5)
ALT - >150 (7-56)
AST - >150 ( 10-40)
GGT - >150 (0-30)
What is the cause of her symptoms?
A- Methotrexate induced hepatitis
B-ibuprofen induced hepatitis
C-Autoimmune hepatitis
D-Viral hepatitis

Patient on methotrexate and prednisolone complaining of mouth ulcer. Treatment asked?


a) Folic acid
b) Folinic acid
c) Stop methotrexate
d) Increase prednisolone

Man with childhood immunization history as complete OPV( 3doses) and other vaccines (
incomplete)
Now going to india for holiday trip. Which vaccines you’d give?
- polio
- MMR
- dTPa
- polio, MMR and dTPa

child with proteinuria 4+ haematuria 1+, puffy, oedema, abdominal distension,


what to test? Short scenario
A. urine c &s
B. ultrasound

AfraTafreeh.com
Get more from: MplusX Qbank (https://member.mplusx.com/)
Facebook Page: (https://www.facebook.com/mplusxqbank/)

C. blood chemistry
D. renal biopsy

Pt with BMI 35, family his of dm 2, obese, FBS is 11. Comes to u. Ur next step.
1. Start metformin
2. Refer to endocrinologist
3. Repeat fbs
4. Rbs
5. Ogtt

Rash with vancomycin immediately during infusion. Diag asked. Pt was already on some
other drugs
1. Idiosyncratic reaction
2. Anaphylaxis
3. Steven johnson
4. Toxic shock

Woman wants to change her will. She is ur old patient. Now coming to u with gifts n asking
ur bdays. Ur step
1. Assess testiment capacity
2. Inform her that she needs to go to legal practitioner

Ur pt for many years. Now comes to u n tells u her personal issues n everything like that.
She says that this is bcz she trusts u. What is most imp to look for in this appointment?
1. Trust
2. Boundaries violation
3. Transference

Pt want to cease smoking. Have tried 4 times but develops severe restlessness agitation n
other symptoms. What is most imp indicator to start nicotine therapy?
1. Copd
2. His 4 attempts of quitting
3. His severe symptoms of withdrawal

Given X-ray of Interior dislocation of shoulder asking which statement is correct.


Loss of sensation upper arm
Loss of sensation triceps
Loss of sensation forearm
No option for deltoid sensation loss

AfraTafreeh.com
Get more from: MplusX Qbank (https://member.mplusx.com/)
Facebook Page: (https://www.facebook.com/mplusxqbank/)

.in a hospital u want to reschedule a seminar at noon which was used to continue at 9 am so that
most people can attend.but that is lunch time.a pharmaceutical company wants to arrange
lunch.what will be the most appro approach?

a.accept the offer but insist that no represensative can’t speak

b.arrange seminer at 9 am

c.tell the employee to bring lunch

d.tell the company to fund for hospital research instead

mother comes with 6mnth child with respiratory tract infection,child got no vaccine and mother dnt
want to give her child vaccine.wht will be ur approach?

a.rfer her to community nurse for councelling

b.report child protection authority

c.tell her its ur duty to give vaccine to the child

d.respect her wish

Young aboriginal male presents to you with insomnia, fear of darkness and seeing “mamu” . He has
been having these symptoms after the death of his mother. Which of the following should be next
step in his treatment?
A-give him benzodiazepine
B-involvement of aboriginal health worker
C-urine drug screen
D-antipsychotic
E-CBT

.36 yr old female comes with swelling of lips n face history of mother and sister have lip and face
swelling cause asked?

a. c1 eastarase deficiency

b. allergy to food

c. drug allergy

Single Mother of 17 year old boy come to u and says that few days back when she came home she
found son wearing his girlfriend underwear....and blames herself for giving birth to the son when his
father left her.....and she coulnot brought him up correctly

AfraTafreeh.com
Get more from: MplusX Qbank (https://member.mplusx.com/)
Facebook Page: (https://www.facebook.com/mplusxqbank/)

1,this is common in children who grow up without father

2,he might b a girl entraped in male body

3,Tranvestism

4, homosexual throughtout his life

5, talk to him.

intern finds the wrong dosage of the warfarin, approached the RMO, but RMO refuses to accept the
mistake saying that the prescription is correct - What to do –
a. inform the Board
b. ask the pharmacist to cross check
c. refer the case every day before the shift
d. inform the director of clinical training

Child comes with excessive vomiting for several times with abdominal pain.No features of diarrhea.
Finger prick test for glucose is normal .What is the immediate danger to him?

A. Cerebral oedema
B. Hypoglycemia
C. Hypokalaemia
D. Hyperkalaemia

Female BMI 20 with repeated treadmill exercise and looks herself every hour in mirror and changes
clothes three times per day, dx:
a. Body dysmorphic disorder
b. Anorexia Nervosa
c. Obsessive compulsive disorder
d. bulimia
e. She is fond of exercising

A man comes with clumsiness of hands and tripping over. Upper limb weakness and fasciculation.
Lower limb muscles also has weakness, increased tendon reflexes. Ankle reflex is not increased in
one side. No wasting. No sensory loss is given. What is the initial investigation?
A. EMG
B. MRI Spine
C. CT scan
D. CSF examination
E. Cervical spine X-ray

4 year old child with cough and nasal discharge. The child is febrile and unwell. There is noisy cough,
intercostal recession. RR-increased, PR-increased. Lungs clear. ENT examination is normal. What is
the most likely diagnosis?

AfraTafreeh.com
Get more from: MplusX Qbank (https://member.mplusx.com/)
Facebook Page: (https://www.facebook.com/mplusxqbank/)

A. Asthma
B. Acute tracheitis
C. Acute laryngotracheobronchitis
D. Acute epiglottis
E. Acute bacterial Pneumonia

A boy come with high fever (38.5) and swollen knee, pain at the medial tubercle of the knee, what
treatment will you give?
a. Penicillin G
b. Penicillin G + gentamycin
c. flucloxacillin
d. erythromycin
e. Ticarcillin + clavulanic

A schizophrenic patient admitted involuntary. He wanted to sue hospital against his admission. He
briefs that God wants him to swim across Pacific Ocean. And he says other people in his religion
have same belief. What's most important justifying his continuing involuntary admission.
a. His poor insight
b. His belief placed him at risk
c. Fixity of his belief
d. Presence of positive psychiatric behavior

Asking common cause of decreased milk production in breastfeeding


a. Less frequent feeding
b. Decrease intake of fluids
c. Spend less time while baby is sucking
d. The baby has a problem in development.
e. positioning

. A lady presented with her baby 5 times in 2 weeks at 10 weeks following delivery. she also
presented at 8 weeks all normal established breastfeeding , baby growing well on 90th percentile.
what relevant info you will ask in her history?
A. her past bad obstetric history
B. her premorbid personality
C. her mood (exact option)
D. history of psychosis
E. Her thoughts about care of baby ( Exact option)

74 years man having check-up for prostate CA, last 2 years PSA was – 1.5ng/ml, now PSA is 3.8ng/ml,
do the 12 slides of biopsy & only one foci show adenocarcinoma, gleason stage is 4, how will u
mange this patient?
a. TURP
b. radical prostatectomy
c. continued surveillance
d. EBRT
e androgen

AfraTafreeh.com
Get more from: MplusX Qbank (https://member.mplusx.com/)
Facebook Page: (https://www.facebook.com/mplusxqbank/)

A mother comes with hemochromatosis in brother. she has 2 kids, aged 2 and 5 years respectively.
Wants advice regarding screening?
A. screen only mother
B. mother and kids
C. ask her to come with husband
D. Screen only children

65 yr old man present with three weeks history of headache & vomiting. He has history of Clark 1
melanoma and surgery 3 years ago. On examination, temperature 37.3 & no other abnormal
findings. CT given. What is the most likely diagnosis? Exact scenario
a.Brain abscess
b. Melanoma metastasis
c. Glioma
d. Cerebral infarct
e. Tuberculous meningitis

Pregnant woman at term.Fetal heart rate 60/min.Emergency LUCS done.Baby's heart rate is
60/min.What antibody will you look for in mother?

Factor V Leiden
Lupus anticoagulant
Anti Ro
Protein C

26 week gestation hypertension


What will you look?
platelets
Fundoscopy
LFT

Fundoscopy picture & asking Dx …

CRAO

Crohn Scenario . After Sulfasalazine , What next?

72 year female come for routine check up. Her mother develop colon cancer at the age of 65 years.
She doesn’t have any bowel symptoms and haven’t done any screening test before. She want to
know her risk? a) Colposcopy 2 yrly
b) 5 yrly cervical cancer screening

AfraTafreeh.com
Get more from: MplusX Qbank (https://member.mplusx.com/)
Facebook Page: (https://www.facebook.com/mplusxqbank/)

c) 5 yearly colon cancer screening


d) mammogram 2 yrly
e) no screening needed

32yr woman with no hx of cancer or any illnesses, no f/h of cancer, comes for screening
a.3yrly fobt
b.2yrly colonoscopy
c.5yrly mammography
d.3 yrly pap smear
e. 2 yrly fobt

A middle aged man who had a history of surgery for achalasia a few years back presents to
you with a complaint of hoarseness for 1 month and dysphagia for 2 weeks. On
examination, vocal cord paralysis was noted and a mass in the thoracic inlet was seen in
imaging. What is the probable diagnosis?
1. cancer of the larynx
2. cancer of the oesophagus
3. cancer of the lung
4. cancer of the thyroid (patient should present lower neck mass)

Poorly controlled Diabetic with mild renal impairment asking about risk for baby due to diabetes
A. IUGR
B. Macrosomia
C. Renal agenesis
D. Intrauterine fetal demise
E. low Birth weight

Acute painful red eye, fixed pupil, IOP 20 mmHg.


a. Timolol
b. Steroid
c. Chloramphenicol
d. Atropine
e. Acetazolamide

Alcoholic patient history of surgery for perforated diverticulum disease , post op 3 day , get
agitation, confused and oxygen 88%(exact) chest examination normal. Most appropriate next invx
A. chest x ray
B. CTPA
C. blood glucose
D. Blood alcohol level
E. Urine creatine and electrolyte

AfraTafreeh.com
Get more from: MplusX Qbank (https://member.mplusx.com/)
Facebook Page: (https://www.facebook.com/mplusxqbank/)

Old patient k/c of COPD and a smoker for more than 20 years. Came with complain of progressive
dyspnea. (SOB, orthop noea, stridor, plethora/
cyanosis, oedema of face and arm, cough, headache, engorged neck veins)
His face was plethoric, edematous and neck veins was engorged till the jaw line. What is diagnosis?
A) Lung CA
B) SVC obstruction
C) Bronchiectasis

40 years old woman with menorrhagia and hysteroscopy. She had a dilatation and curettage three
months ago. Which of the following is the MOST appropriate management plan?
a. nortestosterone
b. levonorgestrel IUCD
c. continuous medroxyprogesterone
d. Tranexamic acid during period
e. Mefanamic acid during period

Farmer came with left swelling in a groin. U did fnac and it shows sqaumous cells. Where is the
lesion
A.Left leg
B.Anus
C.Rectum.
D.testis.
E.penis

18 year old male with bloody diarrhea, you made sigmoidoscopy which shows 10cm friable mucosa.
What is the initial management?
A. Sulfasalazine
B. Rectal Corticosteroid
C. Injection Corticosteroid
D. Lopramide
E. Methotrexate

54yrs old male patient has sudden retrosternal chest pain. Severe vomiting proceed by pain. He has
HT and controlled with Thiazide. On examination, dullness lower left lung and reduce breath sound
on left lower zone of lungs. Which of the following Investigation to reach diagnose?
A) CT chest
B) Gastrograffin swallow
C) USG
D) CXR
E) Echo

Couple come for infertility problem for the last 12 months. On testing examination and tests of the
female are unremarkable. Male has azospermia. And bilateral absence of vas deferens. Which of the
following is most important test before the starting the treatment of infertility?

AfraTafreeh.com
Get more from: MplusX Qbank (https://member.mplusx.com/)
Facebook Page: (https://www.facebook.com/mplusxqbank/)

A. No testing required as they cannot have a child


B. Testing of both male and female for cystic fibrosis
C. Refer for IVF
D. Serum FSH and LH for male
E. Serum Testosterone level

An old man has been taking many medications for some kind of cancer pain. His son found him
unconscious, bringing him to hospital. The patient had pinpoint pupils. He was given naloxone in
ambulance and came back into consciousness. He said the medications he has been taking are
methadone, oxycodone, nicotine patch, paracetamol and another drug (I don’t remember). Which of
the following drug causes this presentation?
A-methadone
B-oxycodone
C-nicotine patch
d. paracetamol

8yr old child brought by his mother , he complained from episode of staring suddenly that occur
along with fidgeting of right hand and movement of right arm head twitching to right side,
sometimes chewing and lip smacking each episode last for 60-90 second then the child remain dizzy
and confused for 1-2 minutes after the episode . These occurs in 3-4 days and then the child back to
his normal activity and behaviour for several weeks . what is the most likely diagnosis
A- Temporal lobe epilepsy
B- Juvenile myoclonic epilepsy
C- Absence seizure
D-Myoclonic seizure
E. Tourette

A woman works at a part-time job (OFFICE CLEANER), and complaints of early morning headache,
frontal & bilateral, dull in character, varying in intensity, she takes paracetamol & ibuprofen, which
only cause relief for 2-3 hours. What is the cause?
A. Drug rebound headache
B. Migraine
C. Tension headache
D. Cerebral tumor
E. Pre-menstrual headache

42 years woman brought to you by her friends who wasn’t go out much for 10 years and stay in
home. But she was enjoying doing gardening and crafting and staying at home. What in history will
help you for her management?
A. School refusal
B. Night terror
C. Alcohol consume
D. Her orientation

Old man presented to you complaining of rest tremor and bradykinesia, also taking respiridone and
he has a known history of visual hallucination and forgetfulness. what is the most likely dx
A. parkinson disease.
B. lewy body dementia
C. alzheimer disease

AfraTafreeh.com
Get more from: MplusX Qbank (https://member.mplusx.com/)
Facebook Page: (https://www.facebook.com/mplusxqbank/)

D. acute delirium.
e. schizophrenia

Which of the following development can be assessed in a 24 mth child who is quite well and brought
by mother?
a. knows 2 pronouns
b. knows name of all colors
c. speaks full sentence
d. know family name
e. know age

Indigenous child with chronic suppurative otitis media presents with chronic ear discharge. Ear swab
done showed existence of pseudomonas aeruginosa organisms. After ear toileting what will u do?
A. Oral Amoxicillin
B. Oral Augmentin
C. Ciprofloxacin ear drop
D. Paramycetin with steroid ear drops
E. Hearing aids

A young man brought to ED after fight in bar in which he hit someone. He is alcoholic, take multiple
drugs and aggressive. What in history will you to know if he has personality disorder?
a. Childhood sexual abuse
b. H/o cruelty to animals in adolescence
c. Drug and alcohol dependence
d. H/o of hitting partner one week back
e. H/o depression in mother

MVA. CT scan (given)- pneumothorax. Breathing is limited due to pain. He is distress, oxygen
saturation is reduced. What is your next appropriate management?
A. Morphine
B. Needle thoracotomy
C. Chest tube drainage

Young man after a quarrel had a fracture of floor of eye what is the most consistent symptom with
that?
a-Conjunctival haemorrhage
b-loss of visual activity
c-anaesthesia around the cheek
d-Cant open the mouth completely
e- Epistaxis

Old lady presented few other features were given with narrow introitus. Biopsy showed lichen
sclerosis management.
A. 0.5% steroid
B. oestrogen cream
C. Surgical removal

AfraTafreeh.com
Get more from: MplusX Qbank (https://member.mplusx.com/)
Facebook Page: (https://www.facebook.com/mplusxqbank/)

An old woman who is widow who seems well in past but since she became a widow and she moves
to the house and start to stay alone one of neighbor had noticed her strange behavior. She starts to
dig some ground in front of the house and when neighbor reached near her and looked at her, she
then aroused him and get aggressive and accused him and all neighbors as imposters. After that she
became calm down and agreed to be seen at the medical clinic. Which one of the following will help
u to get a diagnosis?
A) Thought forms
B) Mood
C) Orientation
D) Delusion
E) Memory

mother borderline and father on antipsychotics, came to u for a sedating med for their baby as
mother says that baby cries a lot and and on examination baby is weak, and lethargic something.
1- Report child protection
2- Give med
3- treat mother
4- treat father
5- good parental education

A sudanese boy had sex with his girlfriend. after a few days he found urethral discharge from his
penis. He has other sexual partners. what specimen will help you make a diagnosis of the STI
a.1st catch urine PCR
b. Mid stream urine pcr
c. first stream urine microscopy and culture
d. Urethral swab gram stain and culture
e. mid-stream urine microscopy and culture

45 yr old lady smoker and htn uses ocps for many years now comes re prescription ,she says doesn't
want to come off because it has controlled her menorrhagia and also says doesn't want to concieve.
What to do now
A. Tell her stop smoking then can give ocps
B. Use implanon
C. Use copper iud
D. Use progestogen only pill
E. mirena

patient with history of claudication, smoke 30 pack cigarette per day, drink alcohol, obese with
diabetic history. he refuse surgery although he was fit for it. He asked you for the appropriate advice
that will improve his symptoms of claudication?
A-Reduce smoking
B-reduce alcohol drinking
C-supervised exercise
D-control his hypercholesterolemia

AfraTafreeh.com
Get more from: MplusX Qbank (https://member.mplusx.com/)
Facebook Page: (https://www.facebook.com/mplusxqbank/)

A lady with BMI of 35. How will you manage her in addition to exercise for long term management?
A. Low Carbohydrate food
B. Lipase inhibitor
C. Diuretics
D. Surgery
E. 4000 kJ/ day

25-year-old boy complain of fatigue. Cousin has blood disorder, require frequent blood transfusion.
On peripheral blood film, there was mentioned target cell found. What will you do next most app?
A. CBC
B. Serum ferritin
C. HB electrophoresis
D. Bone marrow examination

. 1 yr old boy crying and catching his left ear repeatedly. On PE both ear tympanic
membrane red, exudates present on tonsils. Underlying condition for his condition.
a- Acute otitis media
b- Acute tonsillitis
c- Chronic otitis media

Wife with flirtatious behavior with husband's friends and repeated cleaning and was flirting with
doctor too but later became abusive and irritated
A. clozapine

B. lithium

C. diazepam

D. Respiridone

E. Valporate

Child 2 years brought with drowsiness and irritated and dizziness taking sips from the glasses from
Mom’s and dad’s glasses in a party

a. blood alcohol

AfraTafreeh.com
Get more from: MplusX Qbank (https://member.mplusx.com/)
Facebook Page: (https://www.facebook.com/mplusxqbank/)

b. blood glucose

C. mg level

d. blood culture

10-year-old boy who has recent onset of enuresis. the doctor calls his parents for assesment. they
have separated for 3 months. in the interview father tellsyou that he is intending to hide assets from
his ex wife even before the divorce. what should you do?
A. inform the mother

B. do nothing

C. inform registrar at the family court

D. tell father it is because of him the boy was affected

e. Inform police.

A 68 years old women present with 8 time h/o lightheadedness and dizziness.she is on
no regular medication ( no list of medication given in ques,only mentioned no regular
medication )..which one of the following appropriate management for her ?
1)digoxin loading
2)holter monitoring

3)echocardiography (
4)stress echocardiography
5 )angiography

AfraTafreeh.com
Get more from: MplusX Qbank (https://member.mplusx.com/)
Facebook Page: (https://www.facebook.com/mplusxqbank/)

Ecg of atrial fibrillation

Ques 2 )
pt came with palpitation.blood pressure was normal,hypercholesterolaemia was
present…pulse irregular and rate 70b/min .next management?
a.sotalol)
b.amlodipine
c.adenosine
d.warferrin
e amiodarone

Ques 3 )
Ecg look like this ..supraventricular tachycardia
60 yrs old male 10 yrs history of hypertension,feeling dizzy ,and light
headache from this morning when he was doing exercise.smoke 20
cigarettes per day,drinks 2-3 glass of wine most of the night .BP 130/80
,Sitting 120/75 on lying.currently on ibesertan-thiazide 150/12.5 mg
ECG was given(svt) cause of his dizziness.??
A.Dehydration
B.Ischemic heart diseases

AfraTafreeh.com
Get more from: MplusX Qbank (https://member.mplusx.com/)
Facebook Page: (https://www.facebook.com/mplusxqbank/)

C.Hypertension
D.alcholic cardiomyopathy ,
E. MI
Option was almost like these bt one option was diuretics ( making confusion
..though exercise always related to dehydration causing SVT )
Ques 4 )

Ecg of atrial flutter


Scenario was almost like handbook scenario…long standing HTN …dx asked ? atrial
flutter

Ques 5 :ct scan of RCC

renal tumour lower lobe of kidney 5cm, 62 yrs old woman asymptomatic on
accidentally ultrasound finding what you will do next? ( urine analysis mentioned
:rbc > 40 /hpf )
A. Biopsy
B. Total nephrectomy
C. Partial nephrectomy
Ques 7 :

AfraTafreeh.com
Get more from: MplusX Qbank (https://member.mplusx.com/)
Facebook Page: (https://www.facebook.com/mplusxqbank/)

Look like this ct bt rt sided rib # not prominent like this …


CT chest pic MVA with severe chest pain and difficulty breathing due to
pain.decreased breath sounds bilaterally and dullness to percussion.long scenario :
bpnorml ,pulse increased .Asked about the cause of pain?
Hemothorax
Pneumothorax
Pneumomediastinum
Fractured ribs
Ques 8 : pericolic abcess from diverticular disease CT

AfraTafreeh.com
Get more from: MplusX Qbank (https://member.mplusx.com/)
Facebook Page: (https://www.facebook.com/mplusxqbank/)

. CT scan of diverticulitis in elderly woman with scenario of fever, LIF pain, mild
leukocytosis, history of constipation and peri colic abscess found. Most
appropriate next treatment?(CT given)
A. Broad spectrum antibiotics with gut rest
B. Prednisolone
C. CT guided drainage (if more than 4 cm)
D. Laparoscopic surgery
E. sigmoid resection
Ques 9 : ct of metastatic melanoma

An alcoholic man comes to you with complaints of increasing headaches in the


morning. He was operated for a melanoma on his leg 3 years back, CT was
given there was a whitish irregular shadow in left hemisphere. Asking diagnosis:
A. Metastatic Melanoma
B. Haemorrhage

AfraTafreeh.com
Get more from: MplusX Qbank (https://member.mplusx.com/)
Facebook Page: (https://www.facebook.com/mplusxqbank/)

C. Infarct
D. Tumour

Biostat Q : I got a lot of biostat ques ..QUES 10 -Ques 23)


1 ) Rural doctor wants to make study about the relation between admission to
hospital with rota virus infection and birth weight , what’s appropriate ?
a. RCT
b. Case control
c. Cross sectional
d. Cohort
e. Case series

What study will you do to find out association of dental amulgums murcury and
occurrence of dementia in your community
1.Case control;;;;;;;;;;;;;;;;;;;;;;;;;;;;;;;;;;;;;;;ans
2.Cohort
3.RCT
4.Case study
least imp study to look for effectiveness of Rx?
a,case control
b, case series
c, cohort
d,RCT

e, Systemic Review
.study whether smoking causes myocardial infarction?
A.case control
B.cohort
C.Rct
D.cross sectional

Q) Coronary event No coronary event


Aspirin 1 99
No aspirin 2 98
1 2
99 98

Absolute risk reduction


1%;;

2%
50%
100%
Q). Chlamydia stat question data was different ..( ans changed from 2/28 to 3/28
)..asked false negative rate ?

AfraTafreeh.com
Get more from: MplusX Qbank (https://member.mplusx.com/)
Facebook Page: (https://www.facebook.com/mplusxqbank/)

Chlamydia test disease positive Disease negative


Positive 25 4
Negative 3 170

Answer 3/28

. increase report cases of childhood cancer d/t waste production from a factory in
river. Which study?
RCT
B- Cohort
C- Case-control
D- Cross-sectional

A post marketing drug surveillance study of a new heart failure therapy to the market
was carried out on 10,000 subjects who had completed clinical trials. which one of the
following most accurately reflects the information genereated from such a study?

Adverse events profile


comparative therapeutics efficacy
cost benefits trial
cost effectiveness
Drug potency

A pharmaceutical company before the start of the trial got approval from ethical
committee to use a new anti-cancer drug for Ca Pancreas. The ethical committee gave
approval and set the target P value < or = 0.02, for the drug to be superior to other
drugs. At the end of the trial, the company claimed about the efficacy of the new drug to

be superior to all other available drugs in the market and P value at the end of the trial
was = 0.04. What does this P value show here?
New drug is safer as compared to other drugs.

AfraTafreeh.com
Get more from: MplusX Qbank (https://member.mplusx.com/)
Facebook Page: (https://www.facebook.com/mplusxqbank/)

New drug is as useful as other drugs with fewer side effects

new drug is superior to other drugs


New drugs is not to superior to other drugs

Ques 25 : Young indigenous male presents to you with insomnia, fear of darkness
and seeing “mamu” . He has been having these symptoms after the death of his
mother. Which of the following should be next step in his treatment?
a. Give him benzodiazepine
b. Counselling with an indigenous councellor
c. antipsychotic agents
Ques 26 : . Picture of a child with slapped cheek appearance.

Pregnant lady got exposed to this child. Best advice?


A. Assure the virus has no effect on fetus
B. Check maternal serum B19 antibodies
C. Review Hb after one month
D. Tell her to consult immediately if develops rash
Ques 27 .

AfraTafreeh.com
Get more from: MplusX Qbank (https://member.mplusx.com/)
Facebook Page: (https://www.facebook.com/mplusxqbank/)

old lady ,rash ,symmetrical polyarthralgia ,cold finger on nifedipine ANA +.ds
DNA+,ENA-ve ,management ? ( dx :SLE )
Prednisolone

Naproxen
Hydroxychloroquine
Methotrexate
Azathioprine
Ques 28 . 25 yr old female complain of Raynaud's phenomenon in cold,also
having edema of bl feet,,bibasal crepitation on lung bases ,heart on CXR silhute
appearance .ana+, ENA neg.. cause?
a.primary Raynaud's
b.cold agglitonin antibody
c.sle
d.limited sclerosis
Ques 29 :Another ques : limited sclerosis scenario ( rayenauds , ana +ve ,ena _ve )
asking rx ? hydroxicloroquine ans

AfraTafreeh.com
Get more from: MplusX Qbank (https://member.mplusx.com/)
Facebook Page: (https://www.facebook.com/mplusxqbank/)

Ques 30 :

Normal ECG
patient on ramipril only, presented after syncope ,lab value was given, sodium
normal, potassium 5.5,all lab value are normal asked about next management?
a. insulin and glucose
b. haemodialysis
c. rectal resonium
d. Cease ramipril
e. Calcium carbonate
Ques 32 . Colles fracture recall but with different options xray at 6 week followup
was normal and now pt having mild pain and tingling sensations what to do
A. Xray again
B. splint for few days more
C. occupational therapy to check on him
D.exercise analgesics
E. MRI to see ligament
Ques 33 : (new )women with 18 weeks of pregnancy now present with vaginal
bleeding with h/o 3 times loss of fetus at 16/18 weeks .what is the cause of her
fetal loss ?
A)chromosomal abnormalities
B)cervical incompetence
Hormonal imbalance
Ovarian tumor
Valvo vaginitis
Ques 35. An alcoholic, very agitated, brought to the emergency department with
many complaints, claiming if he is not attended to immediately he will jump in
front of a car.his BP ,pulse all is normal .he has h/o of living in street and having

AfraTafreeh.com
Get more from: MplusX Qbank (https://member.mplusx.com/)
Facebook Page: (https://www.facebook.com/mplusxqbank/)

drinks from rejected cane by the side of the street .he also have previous h/o
police custody. What will you check first?
a)blood sugar level)
b)serum magnesium level
c) serum electrolytes (Na+ and K+)
d)urine drug screen
e )other drug level I forgot
( note : no option for serum alcohol ****** )
Ques 36:Boy age 15 present by his mother with complain of abonormal attitude
with his mate .his mate are getting annoyed by his behavior.he has h/o multiple
substance abuse marijuana abuse, multiple cuts.also h/o failed antipsychotic
therapy.most appropriate next management for this boy ?
1.resperidone
2. diazipum
3. fluoxetine

Ques 38. old man wth a mass from xiphisternum to umbilicus.more prominent
when head raised,,having a waist circumference 110cm advice??
Wt reduction
Physiotherapy
Sx
Abdominal binder
Ques 40. 12hr Post op patient of hemicolectomy, agitated, trying to get out of
bed, removing all IV infusion, BP90/60,Pulse 100,SaO2 is 86% in room air. After
giving oxygen by face mask, xray given,,,,,what is next:
Droperidol
Blood glucose level
CT
endotracheal intubate
IV antibiotic
Ques 50:, 7yr old boy ..refusing to go school for few days telling stomach pain in
the mornig….. reasonably well in afternoon,can have lunch properly…. Reluctant
to go to bed until mother come to bed to sleep wth him, become upset when
mother keep him with baby sitter,,,sometimes got angry with 1yr old
brother..father recently got a new job,dx??
Separation anxiety
Generalized anxiety dis

AfraTafreeh.com
Get more from: MplusX Qbank (https://member.mplusx.com/)
Facebook Page: (https://www.facebook.com/mplusxqbank/)

Oppositional defiant
ADHD
Ques 51:BMI-age chart given. 7-year-old boy 30kg wt with BMI 19.5. which of
following is correct?
a. Underweight
b. Overweight
c. Normal wt
d. Obese<
e. BMI is not applicable in children

At 95th percentile line ..so I choose obese


Ques 52 :pt comes with palpitation . on exam BP 160/100, in home bp was in
range of 120/100 to 149/100mm for 2month,what to do to improve health
condition?
Continue exercise and Review in 3month
Review in 1yr
Start enalapril
Echo

AfraTafreeh.com
Get more from: MplusX Qbank (https://member.mplusx.com/)
Facebook Page: (https://www.facebook.com/mplusxqbank/)

24hr BP measurement
Ques 53: . Man with scenario of claudication in left calf. On exam, all peripheral
pulses palpable on right side. Left side all pulses palpable but dorsalis pedis
weak and hardly palpable. Bilateral weakness of ankle reflex and loss of
sensation in foot. side ABI on right side 1, left 0.75. Appropriate investigation?
A. Conventional angiogram
B. CT angiogram
C. CT spine
D. EMG
E. Arterial Doppler
Ques 54:. A preg lady at 39 weeks with no foetal movement or 2 days.CTG
picture normal,no deceleration,with normal foetal heart beat.Management?
a-Admit and observe
b-review after 24hr
c-C/S
d-Ask her to drink juice now and repeat CTG
e..review in next visit

Ques 55: after giving fluid..and resuscitation and defibrilliation done.. no


improvement in rhythm…next management ?
Cpr for 2min
Defibrillation
Dc cardioversion
Bedside USG
Ques 56:
CT given of Renal cell carcinoma. Asking what will increase his risk of DVT if he
undergoes an operation?
a.Nicotine stain of fingers
b. Atrial fibrillation
c.. Bilateral varicose veins
d. BMI 32
e. Spider naevi on chest

Ques 57 : .22yrs pregnant lady at 10 weeks gestation,,no F/H/O, no other


comorbidity,when to screen for GDM ??
FBS now
OGTT now
OGTT at 24-28weeks

AfraTafreeh.com
Get more from: MplusX Qbank (https://member.mplusx.com/)
Facebook Page: (https://www.facebook.com/mplusxqbank/)

No need for screening

Ques 58 : .21 yrs old boy suddenly refued to do university. Have poor
performance in school,Kicked out from football team.. don’t want to meet friends..
stay at home,,fatigue, sleep and eat irregularly,,sometimes agitated,dx??
MDD……
Eating disorder
GAD
Social phobia

Ques 59 :.38 yrs,15 ciggarate smoking ,premenstrual headache,F/H/O breast


carcionoma,most important c/I of OCP
AGE
SMOKING
premenstrual headache
F/H/O breast carcionoma

Ques 60 : 15.Preg lady antinatal visit with anemia ..long lab value given ..most
important info : serum ferritin level given and normal 😊
Hb 9
Mcv low from range ,taking multivitamins already what adv :
Iron infusion
Green leafy vegetable
Hb electrophoresis

Ques 62 :Huge epigastric hernia picture of a 56 year old man which extends from
umbilicus to xiphisternum. He looked morbidly obese and had a waist
circumference of 110 cm Asking for appropriate management for the patient.
Weight loss therapy
Abdominal binder
Herniorraphy with mesh repair
Hernioplasty
Observation and Reassurance

AfraTafreeh.com
Get more from: MplusX Qbank (https://member.mplusx.com/)
Facebook Page: (https://www.facebook.com/mplusxqbank/)

Ques 63 :8 year-old boy has left ear pain. O/E He has 39.1 C fever both left and right
tympanic membranes are bulging and there are suppurative tonsils. what is the
diagnosis ?

A-External otitis

B-Otitis media

C-Tonsillitis

D-Mastoiditis

Ques 65: A 48-year-old farmer smokes 40 cigarettes a day. He goes to the GP


because he has bilateral deadly pain in his buttocks and the backs of his thighs
after walking 100m on a smooth surface and 25m on an incline(down hill) or
rough ground. Dorsalis pedis pulses are bilaterally palpable. Which images will
assist diagnosis?
Spine MRI
CT
Duplex Doppler
Pelvic Angiogram

Ques 67: . 6 days old child,vomiting,mixed jaundice head circumference at 2


percentile, hepatosplenomegaly no eye cataract ,on examination/fundoscopy
there was a pigmentation in retina.next investigation ?
U/s of abdomen
Barium swallow
Karrryo typping
Urine test for virus
Ques 68: Woman on ocps. Presents with 3 episodes of post coital spotting in last
week. No other abnormality. Pap test was normal 2 mnths ago….dx…cervical
ectropion
Whats next
1. Pap test
2. Hpv serology
3. Colposcopy

Ques 70 :baby 6 hours after birth develop jaundice ,whats the cause ?
A )ABO incompetiability

AfraTafreeh.com
Get more from: MplusX Qbank (https://member.mplusx.com/)
Facebook Page: (https://www.facebook.com/mplusxqbank/)

B)physiological jaundice
c)biliary atresia
D)breast milk jaundice

.
Ques 72 : .patient comes for vaccination for cervical cancer . LSIL on pap 6
months ago. Today pap is taken again for testing. what advise for hpv vaccines??
a: No need
b : Depends on hpv serology result
c. Depends on hpv culture result
d: vaccines now
e. depend on today’s pap result

Ques 73 : . lady has done cone biopsy 1 wk back. She developed low grade fever
with mild abdominal pain for last two days. Today the lower abdominal pain is
intensely high with tenderness without gurding per examnation. How to
investigate?
Blood culture
high vaginal swab
low vaginal swab
endocervical swab
Ques 74 : new : a baby brought by mother discharge mixed with blood
…underpant also have blood staining .what investigation you will do ?
a )Sawb from culture
b )examination under G/A
c)report in child protection service

Ques 75 : . You receive a call from the brother of an inpatient who is under your
care. He says thatb his sister has told him that she will commit suicide in the
hospital tomorrow. He also says not to inform his sister that he told you this. What
will be your most appropriate action?
Ask brother to ask his sister to talk about her plans with you
Inform police

AfraTafreeh.com
Get more from: MplusX Qbank (https://member.mplusx.com/)
Facebook Page: (https://www.facebook.com/mplusxqbank/)

Involve legal advisor


Discuss with the patient about what the brother told you
Don’t do anything as the brother has asked to maintain confidentiality

Ques 76 :Ovarian cancer screening question – friend was diagnosed with ovarian
cancer, wants to be screened, you examine her but is normal, what advise do
you give her –
A-ultrasound,
b- reassure,
c- tell her that there is no screening for ovarian cancer

Ques 77 : A 78 year old man with depression being treated with sertraline comes to
you, his wife iscomplaining that his memory has been poor he has become more
withdrawn and he has become more irritable and sometimes very aggressive. You
decide to increase his Sertraline dosage and after 1month review him to see his
symptoms have improved dramatically. What was the Dx?

Major depression
Pseudo dementia(
Vascular dementia
Fronto‐temporal dementia
Ques 78:-a man concerned with his counting rituals, is on d verge of expulsion
frm his job due to his lateness and impaired work performance, asking for d
most appropriate immediate management along with relaxation technique ?
exposure and response prevention
citalopram
olanzapine
diazepam
Ques 79:. A lady comes to you who has CIN 2…on colposcopy lesion is seen
extending beyond vision in endocervical canal. What is your most app next step?
a.Hystrectomy
b.LEEP
c. Cone biopsy
d. Sur

AfraTafreeh.com
Get more from: MplusX Qbank (https://member.mplusx.com/)
Facebook Page: (https://www.facebook.com/mplusxqbank/)

Ques 80 :Indian student, 2 months dysuria, hematuria, frequency. All urine test
normal except RBCs and pus in urine. Urine culture (-). Dx?Jm 249
a)Cystoscopy
b)Renal biopsy
c)Urogram
d)Repeat urinalysis
e)Chlamydia PCR

Ques81 : Patient on fluxetin for depression and nifedipin we for htn.one month ago
Fluoxetine was decrease. Now palpitation and sinus tachycardia. Bp 135/ 75.cause
Drug interactions
Niphidipine
Fluxetin toxicity
Neuroleptic malignant syndrome

:Middle aged woman with pain in right thigh worse at night n morning. But gets
better after 10 mins of walk. Hx of bilateral knee OA. Next Investigation?
X ray right hip…………….ans
Bone scan
X ray lumbosacral spine
MRi
Doppler Usg

Ques 83 :-which modality of treatment is most useful for nasopharyngeal CA?


A.chemotherapy
B.radiotherapy
C.surgery
D.harmonal therapy
E.immunotherapy

Ques 86: what situation not mandatory requirement to report to authority?


A.woman presenting with neck abrasion who admits of being assaulted by partner
B.sewage worker with leptospirosis
C.healthy driver dx epilepsy who needs job to support his family

AfraTafreeh.com
Get more from: MplusX Qbank (https://member.mplusx.com/)
Facebook Page: (https://www.facebook.com/mplusxqbank/)

Ques 88; Patient from Iraq.Gives history of dry cough.No fever not hemoptysis.You
notice pan systolic murmur at apex.Chest x ray given.Diagnosis?
Tb
Mycoplasma Pneumonia
Rheumatic fever
Ca.Bronchus

Ques:89 A patient presented with macroscopic hematuria. 6 months back he was


diagnosed as c-ANCA positive vasculitis causing hematuria and started on
Prednisolone and Cysclophosphamide. The RBCs on examination are 20%
dysmorphic and 80% normal (or maybe opposite not sure). What is the most
appropriate next step?
a. Repeat renal biopsy
b. Stop cyclophosphamide
c. CT abdomen
d. Renal USG
e. Cystoscopy
Ques 90:Restless legs scenerio with normal iron studies asking treatment
Pramipexole…………………ans
woman with steroid use taking risedronate 3months has frequent#Thoracic vertebrae
,what next?
changes to alendronate
increase it
continue
change to IVzolendronic acid……ans
-Ques 91: 3 year old child one day of rash -chicken pox- family will travel in 3
days to new Zealand what do you recommend?
give VZ IG
b.acyclovir
vaccinate the family
nothing
Delay the trip
Ques 92:35 year old woman has a blood pressure of 160/90 mmHg at week 10 of her
first pregnancy. She had recurrent urinary infections in childhood. Her urinalysis shows
protein but no blood. Her blood tests show:
Haemoglobin 109 g/L
Serum urea 7.5 mmol/L n
Serum creatinine 125 μmol/L. inc
What is the most likely cause of her hypertension?
A Chronic glomerulonephritis
B Chronic pyelonephritis

AfraTafreeh.com
Get more from: MplusX Qbank (https://member.mplusx.com/)
Facebook Page: (https://www.facebook.com/mplusxqbank/)

C Essential hypertension
D Pre-eclampsia
E Renal artery stenosis
Ques 93:Middle aged man presents with pain from sole to great toe of
right foot,which awakes him from his sleep. O/E: inflammation from sole to
great toe, non tender????, active and passive movements are painless and
free. No sensory loss, SLR- positive @30 degree. Pedal pulses normal. No
h/o
dm/gout/arthritis. Reflexes are normal. What investigation is most
important for diagnosis?dx:planter fascitis
a) Serum urate
b) MRI lumbar spine
c) FBE………………..initial
angiography
usg
Ques 94:old man plumber and hx of smoking a woerd xray given cant
get diagnosis from xray bilateral infiltrate what is ur
next step in investigation?
a) echo
b) ct chest
c) plueral biopsy
d)ecg

Ques 95:52 y/o female has 1y amenorrhea, 2 years ago pap smear was normal but 4
years ago pap smear was cervical wart, she has had sexual contact after 1 year, during
sex, she had not any discomfort or pain. After that she developed 24 hour vaginal
bleeding. Cause?
vaginal atrophy
cancer cervix…
cancer endometrial

relapse of condyloma…………..
menstrual cycle
Ques 96:18.woman with subtotal thyroidectomy ,in post op room , 12 hrs may b , got
difficult breathing skin red on sutures drain tube collection is few, what to
immediately?
a.remove deep muscle suture
b.remove skin staples

AfraTafreeh.com
Get more from: MplusX Qbank (https://member.mplusx.com/)
Facebook Page: (https://www.facebook.com/mplusxqbank/)

c.intubation
d.oxygen via mask
Ques 97: A middle aged man who had a history of surgery for achalasia a few
years back presents to you with a complaint of hoarseness for 1 month and
dysphagia for 2 weeks. On examination, vocal cord paralysis was noted and a
mass in the thoracic inlet was seen in imaging. What is the probable diagnosis?
1. cancer of the larynx
2. cancer of the oesophagus
3. cancer of the lung
4. cancer of the thyroid

Ques 98:24 weeks pregnant lady with condyloma acumianata treated with
cryotherapy, but the warts appeared(recurrent) again and increasing in size most
appropriate next step in mx? Dx:viral wart
termination of pregnancy
surgical removal of warts
diathermy
pap smear
e )leave until delivery
QUES 99:50yr lady came for screening,father ca colon at 73,mother with cervical
cancer 54,grandmother with breast cancer at 60 or 70,what ll be best screening
adv for her :
Colonoscopy
Colposcopy
Mamography 2yrly………………….ans
Cervical screening test 5yrly

Ques 101:A 55-year-old woman came to your clinic due to progressive tiredness,
associated with occasional arthralgias for 5 months. Upon examination, you note
some skin pigmentation and slight enlargement of the liver. Her father died due to
liver cirrhosis secondary to primary hemochromatosis.
Which of the following is diagnostic result for primary haemochromatosis?
a. Transferrin saturation
b. Increased total iron binding capacity
c. Decreased ferritin
d. Decreased serum iron
e. Increased ESR

AfraTafreeh.com
Get more from: MplusX Qbank (https://member.mplusx.com/)
Facebook Page: (https://www.facebook.com/mplusxqbank/)

Ques 102 : 21years old come with left scrotal pain which wakes him up at
night. No fever, malaise and discharge. On examination, right scrotum and
content is normal. Left scrotum tender, redness, swollen. Cremasteric test
is positive. Pain is reduced initially when left scrotum is elevated. What
invx?
A- Urine C&S
B- Chlamydia PCR
C- Scrotal USG
D- Needle aspiration

Ques 104 :warferrin and gliclazide drug interaction recall v a pt known case of DM she
had an episode of Afib in the past due to which she is on warfarin. Her other medications
were perindopril amlodipine indapamide. He now started on gliclazide. She came to u with
c/o hypoglycemic attacks 2 episodes whch one of following is the culprit drug?
A. Amlodipine
B metoprolol

C indapamide
D perindopril
E warfarin

Ques 105 : ans : phenytoin

Ques 107; Patient went intramedullary wire fixation for tibia


fracture..pain exaggerated on passive dorsiflexion of his big
toe..which management should proceed
A. -review after weeks
B. -put more analgesic
C. -leg elevation
D. -4th option forgot may b invx like xray
E. -stabilize with plaster cast

AfraTafreeh.com
Get more from: MplusX Qbank (https://member.mplusx.com/)
Facebook Page: (https://www.facebook.com/mplusxqbank/)

1. Lady got cracked nipple and painful red and hot swelling on the left breast. What advice to
give?
A. Stop breastfeeding
B. Only breastfeed the baby from right breast
C. Express milk and bottle feed to baby

2. Mother brings her 4 year old boy with the complaints of nocturnal enuresis. He has been
toilet trained since 2.5 years old. During the days he is not wetting but he hasn’t had control
at nights. What is the appropriate regarding this case?
A. Alarm clock
B. Desmopressin
C. Wait for spontaneous remission

3. 35 year old, sexually active, nulliparous woman complaints of two episodes of vaginal
bleeding after sexual intercourse for the past four days. she has never had such problem
before. Her last menstrual period is 24 days ago (didn’t mention more about her menstrual
cycle). What investigation?
A. Pap smear
B. Colposcopy
C. Ultrasound

4. Woman presents with high fever after having cone biopsy 6 days ago. What investigation?
A. Low vaginal swap
B. Endocervical swap
C. Colposcopy
D. Blood culture

5. Women need to do induction of labour (due to severe pre-eclampsia?) and her bishop score
is 2. How to do?
A. PGE1
B. PGE2 after amniotomy
C. PGE2 and wait for 16-24 hours

6. A boy got jelly fish sting at northern queesland and was brought to ER. What is your
immediate action before removing tentacles?
A. Vinegar
B. Hot water
C. Antivenom
D. Pain killer

AfraTafreeh.com
Get more from: MplusX Qbank (https://member.mplusx.com/)
Facebook Page: (https://www.facebook.com/mplusxqbank/)

7. HRT 6 years already. Dexa scan show -1.7. normal ca and vit D level.
A. Alendronate
B. Vit D and calcium
C. Stop HRT

8. Woman already taking alendronate and got hip fracture. She also had other fracture 3
months ago.
A. Rosidronate
B. IV zolandronic
C. Roloxifene
D. Vit D and Ca
E. Strontium

9. What can be find in the anorexia nervosa?


A. Hypertension
B. Tachycardia
C. Vitiligo
D. Ankle oedema

10. 14 year old girl, weight loss over 10kg and BMI 15. Fall in school grades.
A. Episodic fatigue
B. Feeling of guilt
C. Feeling of inferiority
D. Isolation from friends
E. Premorbid personality

11. 28 year old, BMI 35 and smoking, taking OCP for 8 years. Asking advice for ca breast risk.
A. Stop Smoking
B. Weight reduction
C. Low dose oestrogen
D. Self breast examination

12. Alzheimer patient taking donepezil and meboclomide, masturbating and trying to kiss
nurses. Cause?
A. Worsening Alzheimer
B. Donepezil
C. Meboclomide
D. Frontal lobe dementia

AfraTafreeh.com
Get more from: MplusX Qbank (https://member.mplusx.com/)
Facebook Page: (https://www.facebook.com/mplusxqbank/)

13. Patient is diagnosis with cancer. Teary and depressed and requested to let her die.
A. Assess the reason of her request
B. Organize her to join support group of dying people.

14. what is the least affect from lead poisoning


A. neurological abnormalities
B. Abdominal pain
C. Liver abnormalities
D. Behavioral problems

23. 7 day old baby presented with jaundice since 4 days of life, his birth was at term, not
complicated and he is breastfeeding, serum bilirubin was 240 (normal <200) with direct 120(sure) his
liver is palpable 1cm below the costal margin. What is the most appropriate next step?

A. Thyroid function test


B. USG for liver and biliary tree
C. Coomb test
D. Stop breastfeeding and follow-up
E. Phototherapy

23. Aboriginal child with chronic purulent otitis media. Apart from wound toilet, which medication
should be given?

A. Amoxicillin oral
B. Amoxicillin + Clavulanic acid
C. Ciprofloxacin ear-drops

25. A mother asks the GP for advice. Her son has been in hospital with PSGN and she wants to
know when he will be allowed back to school. What should the GP advise?

A. He can return immediately


B. He can return after his infection is gone
C. He can return but should avoid sport and a high protein diet

26. 2yr old boy crying and catching his left ear repeatedly. On PE both ear tympanic memb red ,
exudates present on tonsils.
a- Acute otitis media
b- Acute tonsillitis
c- Chronic otitis media

AfraTafreeh.com
Get more from: MplusX Qbank (https://member.mplusx.com/)
Facebook Page: (https://www.facebook.com/mplusxqbank/)

27. fetal heart rate was 60 and after emergency LUCS, the heart rate remained 60 despite
resuscitation. What maternal investigation will help you with diagnosis.

A. Factor V Leiden
B. Lupus anticoagulant
C. Anti ro antibody
D. Protein C
E. Anticardiolipin autoantibody

28. 4 year old child after cold sores developed pin point lesions on trunk and limbs . lesions were
unblanchable. Labs given. Clinical examination were normal.
Hb- 86g/dl
WCC-normal
Platelets-35*10. Ask treatment. (Dx- childhood ITP)

A. strict bed rest.


B. prednosolone.
C. IV immunoglobulin

29. 18 yr old girl presents with malaise, tiredness for mnths.. on labs, hb 8, microcytic hypochromic,
inr 1.5, calcium 1.9..... what’s the inv to reach the diagnosis. (Many labs were given)
a.hb electrophoresis
b.antigliadin antibodies
c.iron studies
d.serum electrolytes

30. Recurrent candidiasis, response well to drugs which is given by her pharmacist friend. Now
coming to GP for treatment.

A. Vaginal Nystatin
B. Fluconazole
C. clotrimazole

31. 83 years man known case of COPD with 20 cigarettes per day since 18 years of age. Tried to stop
4 times during last year. smoke first cigeratte within 10 mins after wake up. Have strong family
history of smoking. What factor is important indicator to stop smoking or to start nicotine
replacement therapy?
A. Family history
B. Duration of smoking
C. History of cessation
D. Smoking related pulmonary disease
E. First cigeratte time

AfraTafreeh.com
Get more from: MplusX Qbank (https://member.mplusx.com/)
Facebook Page: (https://www.facebook.com/mplusxqbank/)

32. Glaucoma, IOP is 25mmHg, asking for long term treatment?

33. 56 year old man with elongated mass which extends from umbilicus to xiphisternum. Asking for
appropriate management for the patient.
A. Weight reduction
B. physiotherapy

34. A woman after removal of central venous line lady developed facial swelling and swelling around
the neck. What’s the most appropriate investigation?
a. CT chest
b. CT neck angiogram
c. Chest x-ray
d. Neck USG Doppler
e. Echo

35. A patient has taken lots of tablets of venlafaxine. Was on depression treatment. One year back
she was given psychotherapy as she had problem at work (shout at her coworkers). She cant relax at
home after work. After stabilization of the patient, what is the appropriate management?

A. Lorazepam
b. Respiridone
C. Mirtazapine...
D. Haloperidol
E. Sodium valproate

36. Intern of emergency department falling asleep on many nights on his duty. What is initial step to
do?

A. Inform director of emergency department


B. Ask him if he is ok

37. Woman wants to change her will. She is ur old patient. Now coming to u with gifts n asking ur
bdays. Ur step
1. Assess testiment capacity
2. Inform her that she needs to go to legal pr

38. Irish pregnant lady who was born in Australia, comes for first antenatal check up. Her mother has
diabetes. When to check OGTT?

A. OGTT now
B. OGTT at 24-28 weeks
C. OGTT at 20 weeks

AfraTafreeh.com
Get more from: MplusX Qbank (https://member.mplusx.com/)
Facebook Page: (https://www.facebook.com/mplusxqbank/)

D. https://www.health.qld.gov.au/__data/assets/pdf_file/0029/155927/gp-gdm-diagnoses.pdf

39. 21 years old lady with pap smear result of LISL, come for advice about HPV vaccine.
a- Dont vaccine
b- Vaccine now
c- Wait the result of HPV serology
d- wait the result HPV virus type

40. Pregnant lady 35weeks, fetus is with bradycardia and then emergency cesarian was done. After
delivery, baby still with Heart rate of 60bpm. What invx on mother??
a- anti smith
b- lupus
c- anti cardiolipin

d- anti Ro

41. Child with fever and history of sore throat 10 days ago. He had pain in wrist, later
developed arthralgia and swelling in ankle joint. Demarcated rash on the back recently
appeared. What's the most appropriate INITIAL step of management ?
a. ESR
b. Full blood examination
c. throat swab
d. Blood culture
e. ECG

42. Young man with rectal bleeding and found 10cm friable mass on sigmoidoscopy.
Management?
a. steroid
b. sulphasalazine
c. methotrexate

43. A 15year old girl having difficulty in concentrating. She constantly fights with father,
recently stopped ballet class – ask why “what’s the point of all this? “, what will you do?
a. Liase with father
b. safety plan with parents
c. Give ssri
d. CBT
e. Tell school to support her more

AfraTafreeh.com
Get more from: MplusX Qbank (https://member.mplusx.com/)
Facebook Page: (https://www.facebook.com/mplusxqbank/)

44. 12 year old girl at foster care and now presented to you with her foster parents. She is
having difficult time in school in reading and writing. She has a collection of her own toys
and plays with them with her imaginary friend. Her foster cares are irritated because she's
picky at food. Which of the following has immediate danger to her?
a. OCD
b. Sexual abuse
c. Drug abuse
d. Anorexia nervosa
e. Schizophreniform psychosis

46. A man who is intubated and sedated in ICU after operation of cholecystectomy. You are
called to witness for the will he previous signed. What"ll you do?
a. Refuse
b. Sign in front of lawyer
c. Compare the sign from his driving license
d. Inform your medical defense

47. A young lady complains of abdominal pain while urinating, she had menstrual period 9
days ago. She says she also has this kind of pain 4 weeks ago and it lasted for 3 days. What is
the cause?
a. Ectopic pregnancy
b. Ovulation pain
c. Complex ovarian cyst
d. Endometrosis

48. The moon is made of cheese, 'I came by bus’


which one of the following does this pt most likely have?
a. Depression
b. Dissociation
c. Depersonalization
d. Disorganized behavior
e. Delusion

49. Woman forgetfulness, age 65. He forgets where he keeps his things, easily gives up task.
During cognitive assessment she becomes upset and didn’t want to do it. What to check in
mental state examination (MSE)?
a. Orientation
b. Hopelessness

AfraTafreeh.com
Get more from: MplusX Qbank (https://member.mplusx.com/)
Facebook Page: (https://www.facebook.com/mplusxqbank/)

c. Hallucination
d. Insight

51. Young couple with infertility. Female with one child in previous marriage and female
investigations were normal, what in history you will ask the male patient that will be most
useful to guide u to cause of infertility?
a. Do you drink alcohol?
b. Did you get chicken pox infection for once while you were young?
c. Do you use Marijuana?
d. Do you have regular unprotected sex?

52. Oliguria after surgery, no catheter, no fever. What next:


a. Blood culture
b. Bladder scan
c. Eletrolytes

53. COPD with respiratory distress. Sats 88% on oxygen.


Patient known case of chronic lung disease, Heavy smoker, drinks alcohol, presents with
chronic cough, dyspneic and speak in phrases. On examination bilateral wheeze with basal
crackles. What INITIAL investigation should you perfrom?
a. Spirometry
b. CTPA
c. CXR
d. ABG

54. A 55-year-old woman presented with discomfort in her both legs with an urge to move
for last 4 months.She remains awake at night due to creeping and sometimes electric
current-like sensations.Her current medication include paracetamol, oxycodone and
multivitamins.Blood tests show normal renal function, normal electrolytes and slightly low
haemoglobin.
What will you do next?
a. Cease oxycodone
b. Fluoxetine
c. Iron studies
d. Mirtazapine
e. Morphine

AfraTafreeh.com
Get more from: MplusX Qbank (https://member.mplusx.com/)
Facebook Page: (https://www.facebook.com/mplusxqbank/)

56. man recently migrate to Australia married say that people in his country are after him.... bla bla
bla.....what to check in the collateral hx to reach Dx?
A. family hx of MOOD disorder
B. Low sociability in the primary school
C.immigration visa status
D.the truth of his saying
E. Hx of previous physical trauma

57. A wife brought his husband who was not eating and drinking anything for 3 days with severe
melancholic depression. What will you do next?
ECT approval from mental health tribunal
ECT approval under duty of care
ECT approval with consent from wife
ECT approval with consent of two doctors

58. 30 years old woman wants to conceive 12 months of amenorrhea, bmi 30,previously irregular
menstruation 5-6 times a year. Lab values fsh 2(3.3-15), Lh 6(n),prolactin, estrogen, tsh normal.
What will
help in Dx?
A.s.progesteron
B.s.testosterone
C.MRI pituitary
D.Pelvic usg

59. A scenario a man complains severe anal bleeding with pain on defecation. He doesn’t allow to
examine. Pain persists for an hour after defecation
- GTN cream
- Lateral sphincterotomy
- Saline wash and high fiber diet
- Corticosteroid cream

60. Pt with bmi 35, family his of dm 2, obese, fbs is 11. Comes to u. Ur next step.
1. Start metformin
2. Refer to endocrinologist
3. Repeat fbs.
4. Rbs
5. Ogtt

61. A wife brought his husband who was not eating anything for 3 days with melachonic depression.
What you will do next?

AfraTafreeh.com
Get more from: MplusX Qbank (https://member.mplusx.com/)
Facebook Page: (https://www.facebook.com/mplusxqbank/)

A. Ect approval from mental health tribunal


B. Ect approval under duty of care
C. ECT care with consent from the wife
D. ECT approval after discussion with 2 psychiatrists
E. insert NGT and give food

62. Non australian admitted for femur fracture and rib fracture after mva.was driving a stolen car
chased by police some1 killed in accident.he is aggressive argumentative demanding to smoke
.threatens ti asault staf discharge himself and wil complain to embasy.what wil u do acc to last
action?
A.discharge immediately to police custody
B.refer to nearest medical ward with correctional favility
C.allow him to smoke in smokers room safely away from pts
D.insist he use nicotene patch

AfraTafreeh.com

You might also like